Крок 1 - Медицина 2020 2 день (буклет)

1 / 180
На прийом до терапевта прийшов чоловік 37-ми років зі скаргами на періодичні інтенсивні больові напади у суглобах великого пальця стопи та їх припухлість. При аналізі сечі встановлено її різко кислий характер і рожеве забарвлення. З наявністю яких речовин можуть бути пов’язані такі зміни сечі? A 37-year-old man came to see a therapist with complaints of periodic intense pain attacks in the joints of the big toe and their swelling. During the urine analysis, it was found to be sharply acidic nature and pink color. With the presence of what substances can such changes in urine be associated?

Амонієвих солей Ammonium salts

Хлоридів Chlorides

Фосфату кальцію Calcium phosphate

Солей сечової кислоти Uric acid salts

Сульфату магнію Magnesium sulfate

2 / 180
У хворого на цукровий діабет з’явився різкий біль у правій стопі. При огляді великий палець стопи чорного кольору, тканини стопи набряклі, осередки відшарування епідермісу, виділення з неприємним запахом. Яка клініко- морфологічна форма некрозу розвинулася в хворого? A patient with diabetes has a sharp pain in the right foot. On examination, the big toe is black, the tissues of the foot are swollen, cells of epidermal peeling, discharge with an unpleasant smell. What clinical and morphological form of necrosis has developed in the patient?

Гангрена суха Dry gangrene

Іангрена волога Iangrena moisture

Секвестр Sequestration

Пролежень Decubitus bedsores

Інфаркт Heart attack

3 / 180
На розтині трупа жінки 63-х років, що страждала на ревматизм і комбіновану мітральну ваду, виявлено, що стулки мітрального клапана різко потовщені, зрощені між собою, кам’янистої щільності, під час розрізання визначається хрускіт. Який патологічний процес зумовив кам’янисту щільність клапана серця? At the autopsy of a 63-year-old woman suffering from rheumatism and combined mitral valve disease, it was found that the leaflets of the mitral valve were sharply thickened, fused together, rocky density, a crunch is determined during cutting. What pathological process caused the stony density of the heart valve?

Метастатичне звапнення Metastatic calcification

Амілоїдоз Amyloidosis

Дистрофічне звапнення Dystrophic calcification

Фібриноїд Fibrinoid

Метаболічне звапнення Metabolic calcification

4 / 180
У клітину потрапив вірус грипу. Трансляція при біосинтезі вірусного білка в клітині буде здійснюватися: Influenza virus entered the cell. Translation during the biosynthesis of the viral protein in the cell will be carried out:

У клітинному центрі In the cell center

У ядрі In core

На полірибосомах On polyribosomes

На каналах гладенької ендоплазматичної сітки On the channels of the smooth endoplasmic reticulum

У лізосомах In lysosomes

5 / 180
У обстежуваного визначили дихальний об’єм (500 мл), частоту дихання (15 за хвилину), об’єм мертвого простору (100 мл). Скільки повітря пройде у нього за хвилину через альвеоли? The subject's respiratory volume (500 ml), respiratory rate (15 per minute), dead space volume (100 ml) were determined. How much air will pass he has it in a minute through the alveoli?

7500 мл 7500 ml

9000 мл 9000 ml

6000 мл 6000 ml

1500 мл 1500 ml

7400 мл 7400 ml

6 / 180
Експериментатору необхідно якнайшвидше виробити умовний рефлекс у собаки. На базі якого безумовного рефлексу доцільно виробляти умовний? The experimenter needs to develop a conditioned reflex in a dog as soon as possible. On the basis of which unconditioned reflex is it advisable to develop a conditioned one?

Статевий Gender

Травний Digestible

Захисний Protective

Міотатичний Myotatic

Орієнтувальний Indicative

7 / 180
Подразнення правого блукаючого нерва спричинило різке сповільнення атріовентрикулярного проведення. На ЕКГ про це буде свідчити подовження: Irritation of the right vagus nerve caused a sharp slowing of atrioventricular conduction. On the ECG, this will be evidenced by lengthening:

Зубця Т Teeth T

Зубця Р Zubtsia P

Комплексу QRST QRST Complex

Інтервалу RR RR Interval

Інтервалу PQ PQ Interval

8 / 180
Хворому було призначено препарат. Через декілька діб дія препарату значно знизилась і для отримання початкового ефекту потрібно збільшити дозу речовини. Вкажіть назву цього типу зміни дії лікарської речовини: A drug was prescribed to the patient. After a few days, the effect of the drug decreased significantly and to obtain the initial effect, the dose of the substance must be increased. Specify the name of this type of change in the effect of the medicinal substance:

Тахіфілаксія Tachyphylaxis

Кумуляція Cumulative

Лікарська залежність Drug addiction

Звикання Addiction

Ідіосинкразія Idiosyncrasy

9 / 180
У жінки 62-х років розвинулася катаракта (помутніння кришталику) на тлі цукрового діабету. Вкажіть, який тип модифікації білків має місце при діабетичній катаракті: A 62-year-old woman developed a cataract (clouding of the lens) on the background of diabetes. Indicate what type of protein modification occurs in diabetic cataract:

АДФ-рибозилювання ADP-ribosylation

Ілікозилювання Ilicosylation

Фосфорилювання Phosphorylation

Обмежений протеоліз Limited proteolysis

Метилювання Methylation

10 / 180
При кесаревому розтині у хворої 32-х років, внаслідок сильної кровотечі і не-можливості її зупинити, оператор вимушений був видалити матку. За рахунок яких судин здійснюється кровопостачання матки? During a caesarean section of a 32-year-old patient, due to severe bleeding and the inability to stop it, the operator was forced to remove the uterus. Which vessels supply the uterus with blood? ?

Arteria clitoridis Arteria clitoridis

Arteria obturatoria Arteria obturatoria

Arteria sacralis lateralis Arteria sacralis lateralis

Arteria pudenda interna Arteria pudenda interna

Arteria uterina Arteria uterina

11 / 180
У 6-місячної дитини спостерігались часті та сильні підшкірні кровотечі. Призначення синтетичного аналога вітаміну К (вікасола) дало позитивний ефект. В гамма-карбоксилюванні глута- мінової кислоти якого з перерахованих нижче білків згортальної системи крові бере участь цей вітамін? A 6-month-old child had frequent and severe subcutaneous bleeding. The appointment of a synthetic analogue of vitamin K (Vikasol) had a positive effect. In the gamma-carboxylation of glutamic acid, which which of the following proteins of the blood coagulation system is involved in this vitamin?

Протромбіну Prothrombin

Фібриногену Fibrinogen

Фактора Розенталя Rosenthal factor

Антигемофільного глобуліну А Antihemophilic globulin A

Фактора Хагемана Hageman factor

12 / 180
У хворого виявлена серповидноклі- тинна анемія. Заміна якої амінокислоти в поліпептидному ланцюгу НЬ на валін призводить до цього захворювання? The patient was diagnosed with sickle cell anemia. The replacement of which amino acid in the polypeptide chain of Hb by valine leads to this disease?

Треоніну Threonine

Лейцину Leucine

Аспарагінової кислоти Aspartic acid

Аргініну Arginine

Ілутамінової кислоти Ilutamic acid

13 / 180
Під час підйому пішки на 5-й поверх у людини підвищився артеріальний тиск. Причиною є збільшення: While climbing on foot to the 5th floor, a person's blood pressure increased. The reason is the increase:

В’язкості крові Blood viscosity

Хвилинного об’єму крові Minute volume of blood

Об’єму циркулюючої крові Circulating blood volume

Вмісту іонів в плазмі крові Content of ions in blood plasma

Кількості функціонуючих капілярів Numbers of functioning capillaries

14 / 180
В приймальне відділення надійшла дитина 1,5 років з ознаками отруєння нітратами: стійкий ціаноз, задишка, судоми. Утворення якої форми гемоглобіну лежить в основі цих симптомів? A 1.5-year-old child was admitted to the reception department with signs of nitrate poisoning: persistent cyanosis, shortness of breath, convulsions. The formation of which form of hemoglobin is the basis of these symptoms?

Метгемоглобіну Methemoglobin

Карбгемоглобіну Carbhemoglobin

Редукованого гемоглобіну Reduced hemoglobin

Оксигемоглобіну Oxyhemoglobin

Карбоксигемоглобіну Carboxyhemoglobin

15 / 180
Внаслідок росту пухлини в порожнину III шлуночка головного мозку у пацієнта розвиваються вегетативні розлади у вигляді порушення сну, терморегуляції, всіх видів обміну, нецукровий діабет. Подразнення ядер якої ділянки головного мозку викликало ці симптоми? As a result of tumor growth in the cavity of the III ventricle of the brain, the patient develops autonomic disorders in the form of impaired sleep, thermoregulation, all types of metabolism, diabetes insipidus. Irritation of the nuclei of which part of the brain brain caused these symptoms?

Гіпотапамуса Hypotapamus

Довгастого мозку Olongated brain

Покришки середнього мозку Tires of the midbrain

Ніжок мозку Brain stem

Моста Bridges

16 / 180
Хворому призначено препарат дигоксин. Через декілька днів у хворого виявлено ознаки передозування цим препаратом, його вміст у крові значно перевищував верхню межу терапевтичної концентрації. Як називається такий варіант дії лікарських речовин? The patient was prescribed the drug digoxin. After a few days, the patient showed signs of an overdose of this drug, its content in the blood significantly exceeded the upper limit of the therapeutic concentration. What is the name of this variant of the action of the drug substances?

Антагонізм Antagonism

Потенціювання Potentiation

Кумуляція Cumulative

Тахікардія Tachycardia

Звикання Addiction

17 / 180
В результаті травми у чоловіка 47-ми років пошкоджені передні корінці спинного мозку. Відростки яких нейронів пошкоджені? As a result of an injury, a 47-year-old man has damaged the anterior roots of the spinal cord. Which neuron processes are damaged?

Дендрити й аксони чутливих псевдоуніполярних нейронів Dendrites and axons of sensitive pseudounipolar neurons

Дендрити рухових нейронів й аксони ядер бокових стовпів Dendrites of motor neurons and axons of nuclei of lateral columns

Аксони нейронів рухових соматичних та вегетативних ядер Axons of neurons of motor somatic and vegetative nuclei

Дендрити чутливих псевдоуніполярних нейронів Dendrites of sensitive pseudounipolar neurons

Аксони чутливих псевдоуніполярних нейронів Axons of sensitive pseudounipolar neurons

18 / 180
У зоопарк доставлені антилопи з Африки. В їх крові виявлено Trypanosoma brucei gambiense. Чи становлять ці тварини епідеміологічну небезпеку? Antelopes from Africa were brought to the zoo. Trypanosoma brucei gambiense was detected in their blood. Do these animals pose an epidemiological danger?

Небезпека для інших антилоп Danger to other antelopes

Небезпека лише для людини Hazard to humans only

Епідеміологічної небезпеки немає There is no epidemiological danger

Небезпека для домашніх тварин і людини Danger to pets and humans

Небезпека лише для хижаків Danger only for predators

19 / 180
Після травматичного ураження промежини у потерпілого спостерігається нетримання сечі. Який м’яз був травмований? After a traumatic injury to the perineum, the victim has urinary incontinence. What muscle was injured?

Поверхневий поперечний м’яз промежини Superficial transverse perineal muscle

Внутрішній стискач відхідника Internal separator compressor

Зовнішній стискач сечівника External urethral clamp

Сіднично-печеристий м’яз Gluteal-cavernous muscle

Глибокий поперечний м’яз промежини Deep transverse perineal muscle

20 / 180
Хворого з явищами енцефалопатії госпіталізували в неврологічний стаціонар і виявили кореляцію між наростанням енцефалопатії і речовинами, які надходять з кишечника в загальний крово- тік. Яка сполука, що утворюється в кишечнику, може бути причиною такого стану? A patient with symptoms of encephalopathy was hospitalized in a neurological hospital and a correlation was found between the growth of encephalopathy and substances that enter the general bloodstream from the intestines. What compound is formed in intestines, can be the cause of such a condition?

Орнітин Ornithine

Бутират Butyrate

Біотин Biotin

Ацетоацетат Acetoacetate

Індол Indol

21 / 180
До клініки доставили пацієнта 32-х років з масивною крововтратою внаслідок ДТП. Ps- 110/хв., ЧДР- 22 за 2 хв, АТ- 100/60 мм рт.ст. Яка зміна крові з перерахованих буде найбільш характерною через 1 годину після крововтрати? A 32-year-old patient with massive blood loss as a result of a road accident was brought to the clinic. PS- 110/min., CHDR- 22 in 2 min, BP- 100/60 mm Hg. Which of the following blood changes will be the most characteristic 1 hour after blood loss?

Гіповолемія Hypovolemia

Еритропенія Erythropenia

Лейкопенія Leukopenia

Гіпохромія еритроцитів Hypochromia of erythrocytes

Гіпопротеїнемія Hypoproteinemia

22 / 180
Людина отримала електротравму При цьому струм пройшов крізь серцевий м’яз. Які небезпечні порушення в роботі серця можуть виникнути у цій ситуації, що вимагають термінового втручання? A person received an electric shock. At the same time, the current passed through the heart muscle. What dangerous disturbances in the work of the heart can occur in this situation, requiring urgent intervention?

Екстрасистолія Extrasystole

Тахікардія Tachycardia

Брадикардія Bradycardia

Фібриляція шлуночків Ventricular fibrillation

Атріовентрикулярна блокада Atrioventricular block

23 / 180
Після тривалого вживання одного з лікарських препаратів у хворого розвинулась підвищена чутливість до простудних захворювань. Який з вказаних нижче лікарських засобів міг сприяти зниженню імунітету? After long-term use of one of the drugs, the patient developed increased sensitivity to colds. Which of the following drugs could contribute to a decrease in immunity?

Настойка женьшеню Ginseng Tincture

Преднізолон Prednisone

Сустак-форте Sustak-forte

Клофелін Clofelin

Резерпін Reserpin

24 / 180
Дитина 5-ти років після невдалого падіння з дивану скаржиться на різкий біль у шиї, що виникає при найменших рухах. Огляд виявив припухлість, порушення цілісності в ділянці правої ключиці, крепітацію. Що слід ввести в першу чергу для швидкого усунення болю і заспокоєння дитини? A 5-year-old child after an unfortunate fall from the sofa complains of a sharp pain in the neck that occurs with the slightest movement. The examination revealed swelling, a violation of integrity in the area of the right clavicle , crepitation. What should be introduced first of all to quickly eliminate pain and calm the child?

Парацетамол Paracetamol

Кодеїн Codeine

Анальгін Analgin

Промедол Promedol

Діазепам Diazepam

25 / 180
Чоловіку 70-ти років, який страждає на хворобу Паркінсона, був призначений препарат леводопа. Через тиждень стан хворого значно покращився. Який механізм дії лікарського засобу? A 70-year-old man suffering from Parkinson's disease was prescribed levodopa. After a week, the patient's condition improved significantly. What is the mechanism of action of the drug?

Активація енкефалінової системи Enkephalin system activation

Іальмування серотонінергічної системи Ialmation of the serotonergic system

Іальмування холінергічної системи Ialmation of the cholinergic system

Активація дофамінової системи Activation of the dopamine system

Іальмування гістамінергічної системи Ialmation of the histaminergic system

26 / 180
Жінка 37-ми років звернулася до гінеколога з приводу запального процесу піхви, який супроводжується свербінням та пінистими виділеннями. Бакана- ліз встановив наявність трихомонадної інфекції. Який засіб найбільш ефективний у даному випадку? A 37-year-old woman consulted a gynecologist about an inflammatory process of the vagina, which is accompanied by itching and foamy discharge. Bacchanalization revealed the presence of a trichomonas infection. What is the most effective remedy in this case?

Ампіцилін Ampicillin

Метронідазол Metronidazole

Ністатин Nystatin

Нітроксолін Nitroxoline

Еритроміцин Erythromycin

27 / 180
При обстеженні хворого на гемофілію виявлено зміну деяких показників крові. Яка з перерахованих ознак відповідає цьому захворюванню? When examining a patient with hemophilia, a change in some blood parameters was detected. Which of the listed signs corresponds to this disease?

Час згортання крові вповільнений Blood clotting time is slow

Еозинофілія Eosinophilia

Афібриногенемія Afibrinogenemia

Тромбоцитопенія Thrombocytopenia

Еритроцитоз Erythrocytosis

28 / 180
У хворого, який страждав на хронічний алкоголізм і цироз печінки, розвинулася профузна кровотеча з варикозно розширених вен стравоходу, в результаті чого настала смерть. На аутопсії печінка дрібногорбиста, зменшена в розмірах, щільна, жовтуватого кольору. Під час гістологічного дослідження кріостатних зрізів печінки (забарвлення гематоксиліном і еозином) в гепатоцитах виявляються великі оптично порожні вакуолі, в яких міститься речовина, що забарвлюється в чорний колір при застосуванні осмієвої кислоти. Оптично порожні вакуолі гепатоцитів - це: A patient suffering from chronic alcoholism and cirrhosis of the liver developed profuse bleeding from varicose veins of the esophagus, resulting in death. At autopsy, the liver was slightly humped, reduced in size, dense, yellowish in color. During histological examination of cryostat sections of the liver (hematoxylin and eosin staining), large optically empty vacuoles are found in hepatocytes, which contain a substance that turns black when osmic acid is used. Optically empty vacuoles of hepatocytes are :

Алкогольний гіалін (тільця Меллорі) Alcohol hyaline (Mallory bodies)

Великокрапельна жирова дистрофія Large droplet fatty dystrophy

Гідропічна дистрофія Hydropic dystrophy

Псевдовакуолі гіалоплазми Pseudovacuoles of hyaloplasm

Включення гіаліну Inclusion of hyaline

29 / 180
Дослідник в ході мікроскопічного і електронно-мікроскопічного вивчення печінки звернув увагу, що деякі окремо розташовані клітини розпалися на дрібні фрагменти, оточені мембраною. У деяких з них є органели, інші включають в себе фрагменти ядра, що розпало-ся. Запальна реакція навколо була відсутня. Він розцінив ці зміни як прояв: In the course of a microscopic and electron-microscopic study of the liver, the researcher noticed that some separately located cells broke up into small fragments surrounded by a membrane. Some of them have organelles, others include fragments of a disintegrated nucleus. There was no surrounding inflammatory reaction. He regarded these changes as a manifestation of:

Апоптозу Apoptosis

Дистрофії Dystrophies

Некрозу Necrosis

Атрофії Atrophies

Гіпоплазії Hypoplasias

30 / 180
У гістопрепараті визначається орган, що складається з сірої та білої речовини. Сіра речовина розташовується на периферії й має 6 шарів: молекулярний, зовнішній зернистий, пірамідний, внутрішній зернистий, гангліонарний і шар поліморфних клітин. Визначте утворення, якому належать дані морфологічні ознаки: In the histopreparation, an organ consisting of gray and white matter is determined. The gray matter is located on the periphery and has 6 layers: molecular, external granular, pyramidal, internal granular, ganglion and a layer of polymorphic cells. Define the formation to which these morphological features belong:

Кора великих півкуль Cortex of large hemispheres

Спинний мозок Spinal cord

Мозочок Cerebellum

Довгастий мозок Olongate brain

Спинномозковий вузол Spinal node

31 / 180
Машиною швидкої допомоги доставлено до приймального відділення лікарні людину з гострим отруєнням морфіном. Який із найбільш специфічних антагоністів необхідно застосувати в цьому випадку? A person with acute morphine poisoning was brought to the hospital by ambulance. Which of the most specific antagonists should be used in this case?

Дипіроксим Dipiroxime

Пентазоцин Pentazocine

Омнопон Omnopon

Метацин Metacin

Налоксон Naloxone

32 / 180
Хворий переніс повторний інтраму- ральний інфаркт міокарда. Після лікування та реабілітації був виписаний у задовільному стані під нагляд дільничного терапевта. Через 2 роки загинув у автомобільній катастрофі. Який характер патологічного процесу в міокарді було встановлено на розтині? The patient suffered a repeated intramural myocardial infarction. After treatment and rehabilitation, he was discharged in a satisfactory condition under the supervision of a district therapist. He died in a car accident 2 years later. What kind of character pathological process in the myocardium was established at autopsy?

Великовогнищевий кардіосклероз Multifocal cardiosclerosis

Дрібновогнищевий кардіосклероз Microfocal cardiosclerosis

Некроз Necrosis

Гіперплазія Hyperplasia

Атрофія Atrophy

33 / 180
Хвора на бронхіальну астму приймала протягом 3-х місяців преднізолон у таблетках. Внаслідок значного покращення стану раптово припинила його прийом. Розвиток якого ускладнення ви- сокоймовірний у цьому випадку? A patient with bronchial asthma took prednisone in tablets for 3 months. Due to a significant improvement in her condition, she suddenly stopped taking it. Which complication is most likely to develop in this case?

Гіпотонії Hypotonia

Ожиріння верхньої половини тулуба Obesity of the upper half of the body

Синдрому відміни Withdrawal syndrome

Шлункової кровотечі Gastric bleeding

Синдрому Іценка-Кушинга Itsenko-Cushing syndrome

34 / 180
У клініку госпіталізований хворий з отруєнням. Встановлено, що в печінці порушені механізми детоксикації. Які з органел гепатоцитів в першу чергу обумовили цей стан? A patient with poisoning was hospitalized in the clinic. It was established that the detoxification mechanisms in the liver are disturbed. Which of the hepatocyte organelles caused this condition in the first place?

Гранулярна ендоплазматична сітка Granular endoplasmic reticulum

Мітохондрїї Mitochondria

Рибосоми Ribosomes

Комплекс Іольджі Iolji Complex

Агранулярна ендоплазматична сітка Agranular endoplasmic reticulum

35 / 180
На земній кулі існують території (біохімічні провінції) частина населення яких страждає на ендемічний зоб. Дефіцит якого біоелемента в грунті, воді та харчових продуктах викликає це захворювання? There are territories (biochemical provinces) on the globe where part of the population suffers from endemic goiter. The deficiency of which bioelement in the soil, water and food products causes this disease?

Мідь Copper

Залізо Iron

Йод Iodine

Кобальт Cobalt

Цинк Zinc

36 / 180
У 12-річного хлопчика в сечі виявлено високий вміст усіх амінокислот аліфатичного ряду. При цьому відзначена найвища екскреція цистину та цистеїну. Крім того, УЗД нирок показало наявність у них каменів. Виберіть можливу патологію: In a 12-year-old boy, a high content of all amino acids of the aliphatic series was detected in the urine. At the same time, the highest excretion of cystine and cysteine was noted. In addition, ultrasound of the kidneys showed the presence of stones. Select a possible pathology:

Алкаптонурія Alkaptonuria

Цистинурія Cystinuria

Цистит Cystitis

Хвороба Хартнупа Hartnup's disease

Фенілкетонурія Phenylketonuria

37 / 180
Внаслідок захворювання нирок у пацієнта відзначаються набряки. В аналізах сечі визначається масивна протеїну- рія. Який механізм є основним у виникненні набряків у такого пацієнта? As a result of kidney disease, the patient has edema. Massive proteinuria is detected in urine tests. What mechanism is the main cause of edema in such a patient?

Зниження фільтраційного тиску в нирках Decreased filtration pressure in the kidneys

Підвищення осмотичного тиску плазми крові Increased blood plasma osmotic pressure

Зниження онкотичного тиску тканин Reduction of tissue oncotic pressure

Зниження онкотичного тиску лімфи Decreased lymph oncotic pressure

Зниження онкотичного тиску плазми крові Reduction of oncotic pressure of blood plasma

38 / 180
У пацієнта з гнійничковими ураженнями шкіри виділено збудник, який на кров’яному агарі утворює жовті колонії округлої форми, середніх розмірів, оточені зоною гемолізу. В мазках з колоній містяться коки, розташовані скупченнями неправильної форми, Гр+. Ви-ділена культура є оксидазо- та каталазопозитивною, ферментує маніт, синтезує плазмокоагулазу Якому виду з нижче- перерахованих відповідає виділений збудник? In a patient with pustular lesions of the skin, the pathogen was isolated, which on blood agar forms yellow colonies of a rounded shape, of medium size, surrounded by a zone of hemolysis. The smears from the colonies contain cocci, arranged in clusters of irregular shape, Gr+. The isolated culture is oxidase- and catalase-positive, ferments mannitol, synthesizes plasma coagulase. Which type of the following does the isolated pathogen correspond to?

Staphylococcus aureus Staphylococcus aureus

Staphylococcus saprophyticus Staphylococcus saprophyticus

Staphylococcus epidermidis Staphylococcus epidermidis

Streptococcus agalactiae Streptococcus agalactiae

Streptococcus pyogenes Streptococcus pyogenes

39 / 180
У хворого на цукровий діабет виникла значна спрага, дисфагія та порушення психічної діяльності. Який тип розладів водно-електролітного балансу характеризує поява вказаних ознак? A patient with diabetes has significant thirst, dysphagia, and impaired mental activity. What type of water-electrolyte balance disorders is characterized by the appearance of these symptoms?

Дегідратація гіпоосмотична Hyposmotic dehydration

Гідратація гіпоосмотична Hyposmotic hydration

Дегідратація гіперосмотична Hyperosmotic dehydration

Дегідратація ізоосмотична Dehydration isosmotic

Гідратація ізоосмотична Hydration isosmotic

40 / 180
Потерпілий обробляв рослини розчином речовини з інсектицидною дією без індивідуальних засобів захисту. Через деякий час у нього почалося сильне виділення слини, поту, сліз, біль у животі, понос. При огляді виявлено міоз. Речовина, що викликала отруєння, належить до групи: The victim treated plants with a solution of a substance with an insecticidal effect without personal protective equipment. After some time, he began to have a strong secretion of saliva, sweat, tears, abdominal pain, diarrhea. The examination revealed miosis. The substance that caused the poisoning belongs to the group:

Антихолінестеразні засоби Anticholinesterase drugs

Солі міді Copper salts

Нітрати Nitrates

Органічні сполуки хлору Organic chlorine compounds

Н-холіноміметики H-cholinomimetics

41 / 180
Пацієнт 64-х років має гостру серцеву недостатність, артеріальний тиск - 80/60 мм рт.ст., добовий діурез - 530 мл, істотно збільшену концентрацію сечовини та креатиніну в крові. Назвіть патогенетичний механізм розвитку азотемії та олигурії: A 64-year-old patient has acute heart failure, blood pressure - 80/60 mm Hg, daily urine output - 530 ml, significantly increased concentration of urea and of creatinine in the blood. Name the pathogenetic mechanism of the development of azotemia and oliguria:

Зменшення об’єму циркулюючої крові Decreasing the volume of circulating blood

Збільшення продукції вазопресину Increased production of vasopressin

Спазм приносних артеріол клубочка Spasm of afferent glomerular arterioles

Гіпернатріємія Hypernatremia

Зменшення фільтраційного тиску в нирках Reduction of filtration pressure in the kidneys

42 / 180
Людина довгий час проживала в умовах високогір’я. Які зміни в кровоносній системі будуть у неї? A person has lived in the highlands for a long time. What changes will he have in his circulatory system?

Збільшення кількості лейкоцитів Increase in the number of white blood cells

Вповільнення пульсу Slow down pulse

Збільшення діаметра кровоносних судин Increasing diameter of blood vessels

Збільшення кількості гемоглобіну Increase in hemoglobin

Зниження кількості лейкоцитів Decreasing the number of white blood cells

43 / 180
При підйомі в гори у альпініста розвинулась ейфорія, яка замінилася головним болем, запамороченням, серцебиттям, задишкою, що перейшла в апное. Яке порушення кислотно-лужної рівноваги розвинулось в даному випадку? While climbing the mountains, the climber developed euphoria, which was replaced by headache, dizziness, palpitations, shortness of breath, which turned into apnea. What acid-base balance disorder developed in in this case?

Видільний алкалоз Excretory alkalosis

Газовий алкалоз Gas alkalosis

Негазовий ацидоз Nongaseous acidosis

Негазовий алкалоз Nongaseous alkalosis

Іазовий ацидоз Iasic acidosis

44 / 180
У дівчинки 6-ти років пастозність повік, губ, шиї, слизової оболонки язика виникла після того, як вона з’їла апельсин. Раніше на апельсини виникали висипання на шкірі, свербіння. Який патогенетичний механізм є провідним у розвитку набряку у дівчинки? A 6-year-old girl developed pasty of the eyelids, lips, neck, and mucous membrane of the tongue after she ate an orange. Previously, oranges caused a rash on skin, itching. What pathogenetic mechanism is leading in the development of edema in a girl?

Підвищення проникності капілярів Increasing permeability of capillaries

Порушення лімфовідтоку Disruption of lymphatic drainage

Підвищення онкотичного тиску тканинної рідини Increased oncotic pressure of tissue fluid

Підвищення гідростатичного тиску крові в капілярах Increased hydrostatic blood pressure in capillaries

Зниження онкотичного тиску крові Reduction of blood oncotic pressure

45 / 180
На прийом до лікаря прийшов пацієнт дуже високого зросту, з довгими товстими пальцями рук, великою нижньою щелепою і відвислою нижньою губою. Підвищену секрецію якого гормону якої залози можна припустити? A very tall patient, with long thick fingers, a large lower jaw and a drooping lower lip, came to the doctor for an appointment. Increased secretion of which hormone of which gland can be assumed?

Соматотропного гормону передньої частки гіпофіза Somatotropic hormone of the anterior lobe of the pituitary gland

Антидіуретичного гормону задньої частки гіпофіза Antidiuretic hormone of the posterior lobe of the pituitary gland

Гормонів щитоподібної залози Thyroid hormones

Гонадотропного гормону передньої частки гіпофіза Gonadotrophic hormone of the anterior lobe of the pituitary gland

Гормонів наднирників із групи глюко- кортикоїдів Glucocorticoid adrenal hormones

46 / 180
Під час гістологічного дослідження щитоподібної залози, видаленої в ході операції, виявлена деструкція й атрофія фолікулів, дифузна лімфоцитарна інфільтрація з формуванням лімфоїдних фолікулів в стромі. До якої групи захворювань належить такий тиреоїдит? During the histological examination of the thyroid gland removed during surgery, destruction and atrophy of the follicles, diffuse lymphocytic infiltration with the formation of lymphoid follicles in the stroma were found. To which group of diseases does it belong such thyroiditis?

Аутоімунних Autoimmune

Викликаних фізичними факторами Caused by physical factors

Вірусних Virals

Бактеріальних Bacterial

Інфекційно-алергічних Infectious-allergic

47 / 180
Дівчинка 4-х років на 3-ю добу від початку захворювання на дифтерію померла від справжнього крупу На ауто- псії слизова оболонка гортані, трахеї та бронхів потовщена, набрякла, тьмяна, вкрита сіруватою плівкою, що легко відокремлюється. Визначити вид ексудативного запалення гортані: A 4-year-old girl died of true croup on the 3rd day after the onset of diphtheria. At autopsy, the mucous membrane of the larynx, trachea, and bronchi is thickened and swollen , dull, covered with a grayish film that is easily separated. Determine the type of exudative inflammation of the larynx:

Серозне Serious

Катаральне Catarrhal

Змішане Mixed

Фібринозне Fibrinous

Гнійне Purulent

48 / 180
У хворого внаслідок отруєння сулемою розвинулася гостра ниркова недостатність, перебіг якої включав 4 стадії: перша - початкова, друга - оліго-, анурії, четверта - одужання. Як називається третя стадія гострої ниркової недостатності? As a result of sulema poisoning, the patient developed acute kidney failure, the course of which included 4 stages: the first - initial, the second - oligo-, anuria, the fourth - recovery. What is the name the third stage of acute renal failure?

Ішемічна Ischemic

Іемодинамічна Iemodynamic

Метаболічна Metabolic

Патохімічна Pathochemical

Поліурична Polyuric

49 / 180
Злоякісна гіперхромна анемія хвороба Бірмера виникає внаслідок нестачі вітаміну В12. Який біоелемент входить до складу цього вітаміну? Malignant hyperchromic anemia Birmer's disease occurs as a result of a lack of vitamin B12. What bioelement is part of this vitamin?

Залізо Iron

Молібден Molybdenum

Магній Magnesium

Кобальт Cobalt

Цинк Zinc

50 / 180
У хворого час від часу суттєво знижується вміст гемоглобіну і еритроцитів в крові, з’являється жовтяниця. З анамнезу з’ясовано, що ці напади завжди виникають після вживання кінських бобів. Яка з форм анемії має місце у даного хворого? From time to time, the patient has a significant decrease in hemoglobin and erythrocytes in the blood, jaundice appears. From the anamnesis, it was found that these attacks always occur after consuming horseradish beans. What form of anemia does this patient have?

Спадкова ферментопатія Hereditary enzyme disease

Спадкова мембранопатія Hereditary membranopathy

Залізодефіцитна анемія Iron deficiency anemia

Спадкова гемоглобінопатія Hereditary hemoglobinopathy

Набута гемолітична анемія Acquired hemolytic anemia

51 / 180
На розтині тіла померлого виявлено, що вся права легеня збільшена, щільна, на плеврі нашарування фібрину, на розрізі тканина сірого кольору, з якої стікає каламутна рідина. Для якого захворювання легенів характерна така картина? At the autopsy of the dead body, it was found that the entire right lung was enlarged, dense, on the pleura layers of fibrin, on the cross section, the tissue was gray in color, from which a cloudy liquid was flowing. For which lung disease is characterized by such a picture?

Фіброзивний альвеоліт Fibrosing alveolitis

Крупозна пневмонія Croup pneumonia

Інтерстиціальна пневмонія Interstitial pneumonia

Вогнищева пневмонія Focal pneumonia

Іангрена легені Iangrena lung

52 / 180
На розтині хворого, який багато років працював на шахті і помер від хронічної легенево-серцевої недостатності, виявлено, що легені малоповітряні, значно ущільнені, склерозовані, верхівки емфізематозно змінені, поверхня сіро- чорного кольору, на розрізі тканина легенів аспідно-чорного кольору. Від якої хвороби настала смерть? At the autopsy of a patient who worked in a mine for many years and died of chronic pulmonary heart failure, it was found that the lungs were hypoaerated, significantly compacted, sclerosed, and the apices were emphysematously changed , the surface is gray-black, the lung tissue is aspid-black in cross-section. What disease caused death?

Антракоз Anthracosis

Талькоз Talkoz

Асбестоз Asbestosis

Силікоз Silicosis

Алюміноз Aluminoz

53 / 180
У потерпілого травма внаслідок прямого удару по внутрішній поверхні середньої третини гомілки. Перелом якого анатомічного утворення найбільш імовірний? The victim suffered an injury as a result of a direct blow to the inner surface of the middle third of the lower leg. Which anatomical formation is the most likely fracture?

Дистальний епіфіз малогомілкової кістки Distal epiphysis of fibula

Дистальний епіфіз великогомілкової кістки Distal epiphysis of tibia

Проксимальний епіфіз малогомілкової кістки Proximal epiphysis of the fibula

Діафіз великогомілкової кістки Tibia diaphysis

Проксимальний епіфіз великогомілкової кістки Proximal tibial epiphysis

54 / 180
При мікроскопічному дослідженні легені недоношеної дитини виявлено спадання стінок альвеол через відсутність сурфактанту. Вкажіть, з порушенням розвитку яких клітин стінки альвеоли це пов’язано: During a microscopic examination of the lungs of a premature child, the collapse of the alveolar walls due to the lack of surfactant was detected. Indicate the development disorder of which cells of the alveolar wall this is related to:

Секреторних клітин Secretory cells

Фібробластів Fibroblasts

Альвеолярних макрофагів Alveolar macrophages

Альвеолоцитів II типу Type II alveolocytes

Альвеолоцитів І типу Type I alveolocytes

55 / 180
Хворий 70-ти років, який страждав на цукровий діабет та переніс інфаркт міокарда, помер при явищах прогресуючої серцево-судинної недостатності. На розтині ціанотична індурація селезінки та нирок, бура індурація легень та мускатна печінка. Який вид порушення кровообігу обумовив зміни внутрішніх органів?  A 70-year-old patient, who suffered from diabetes and suffered a myocardial infarction, died of progressive cardiovascular failure. At autopsy, cyanotic induration of the spleen and kidneys, brown induration of the lungs and nutmeg liver. What type of circulatory disturbance caused the changes in the internal organs?

Загальна артеріальна гіперемія після анемії General arterial hyperemia after anemia

Артеріальна ішемія в результаті перерозподілу крові Arterial ischemia as a result of redistribution of blood

Загальна гостра венозна гіперемія General acute venous hyperemia

Місцева хронічна венозна гіперемія Local chronic venous hyperemia

Загальна хронічна венозна гіперемія General chronic venous hyperemia

56 / 180
На слизовій оболонці мигдаликів та м’якого піднебіння виявляються білувато-сірого кольору плівки, які щільно з’єднані з підлеглою тканиною. При спробі зняти плівку на її місці виникає глибокий дефект тканини. Визначити патологічний процес, який виник на слизовій оболонці мигдаликів та м’якого піднебіння: On the mucous membrane of the tonsils and soft palate, there are whitish-gray films that are tightly connected to the underlying tissue. When you try to remove the film, there is a deep tissue defect. Determine the pathological process that occurred on the mucous membrane of the tonsils and soft palate:

Дифтеритичне запалення Diphtheritic inflammation

Гнійне запалення Purulent inflammation

Серозне запалення Serous inflammation

Змішане запалення Mixed inflammation

Крупозне запалення Croupous inflammation

57 / 180
.3 метою ранньої діагностики вагітності досліджується сеча жінки. З’явлення яких гормонів в сечі свідчить про ймовірну вагітність? .3 for the purpose of early diagnosis of pregnancy, a woman's urine is examined. The appearance of which hormones in the urine indicates a probable pregnancy?

17-бета-естрадіол 17-beta-estradiol

Хоріонічний гонадотропін chorionic gonadotropin

Тестостерон Testosterone

Естріол Estriol

Прогестерон Progesterone

58 / 180
Людина знепритомніла у салоні автомобіля, де тривалий час очікувала приятеля при ввімкненому двигуні. В крові у неї знайдено сполуку гемоглобіну. Яку саме? A person fainted in the passenger compartment of a car, where he had been waiting for a friend for a long time with the engine running. Hemoglobin compound was found in his blood. Which one exactly?

Карбоксигемоглобін Carboxyhemoglobin

Карбгемоглобін Carbhemoglobin

Дезоксигемоглобін Deoxyhemoglobin

Оксигемоглобін Oxyhemoglobin

Метгемоглобін Methemoglobin

59 / 180
При обстеженні людини встановлено, що хвилинний об’єм серця дорівнює 3500 мл, систолічний об’єм - 50 мл. Якою є у людини частота серцевих скорочень? When examining a person, it was found that the minute volume of the heart is 3500 ml, the systolic volume is 50 ml. What is the heart rate of a person?

50 скорочень за хвилину 50 cuts per minute

60 скорочень за хвилину 60 cuts per minute

70 скорочень за хвилину 70 cuts per minute

90 скорочень за хвилину 90 cuts per minute

80 скорочень за хвилину 80 cuts per minute

60 / 180
У людини внаслідок хронічного захворювання печінки суттєво порушена її білковосинтезуюча функція. До зменшення якого параметру гомеостазу це призведе? Due to a chronic liver disease, a person's protein-synthesizing function is significantly impaired. Which parameter of homeostasis will this lead to a decrease?

Осмотичний тиск Osmotic pressure

pH pH

Щільність крові Blood density

Онкотичний тиск плазми крові Oncotic pressure of blood plasma

Іематокритний показник Hematocrit index

61 / 180
Унітіол є антидотом і застосовується, зокрема, при отруєннях солями важких металів. Як називається такий тип взаємодії лікарських речовин? Unithiol is an antidote and is used, in particular, for poisoning with salts of heavy metals. What is the name of this type of drug interaction?

Фізичний антагонізм Physical Antagonism

Неконкурентний антагонізм Non-competitive antagonism

Фізіологічний антагонізм Physiological antagonism

Хімічний антагонізм Chemical antagonism

Синергоантагонізм Synergoantagonism

62 / 180
Хворому на туберкульоз в комплексній терапії призначено препарат - похідне гідразиду ізонікотинової кислоти. Визначити цей препарат: A tuberculosis patient in complex therapy is prescribed a drug - a derivative of isonicotinic acid hydrazide. Identify this drug:

Стрептоміцину сульфат Streptomycin sulfate

Канаміцин Kanamycin

Рифампіцин Rifampicin

Ізоніазид Isoniazid

Цефалоридин Cefaloridine

63 / 180
Робітник тваринницької ферми гостро захворів і при наростаючих явищах інтоксикації помер. На розтині встановлено: селезінка збільшена, в’яла, на розрізі темно-вишневого кольору, зішкріб пульпи рясний. М’які мозкові оболонки на склепінні та основі мозку набряклі, просякнуті кров’ю, мають темно-червоний колір ('шапочка кардинала'). Мікроскопічно: серозно- геморагічне запалення оболонок і тканин головного мозку з руйнуванням стінок дрібних судин. Поставте діагноз: A worker on a livestock farm became acutely ill and died due to increasing symptoms of intoxication. The autopsy revealed: the spleen was enlarged, flaccid, dark cherry-colored on section, scraping of the pulp was abundant . The soft meninges on the vault and base of the brain are swollen, soaked with blood, have a dark red color ('cardinal's cap'). Microscopically: serous-hemorrhagic inflammation of the membranes and tissues of the brain with destruction of the walls of small vessels. Make a diagnosis:

Туляремія Tularemia

Холера Cholera

Сибірка Anthrax

Бруцельоз Brucellosis

Чума Plague

64 / 180
Під час розтину тіла жінки, яка померла внаслідок пухлинної дисемінації муцинозної цистаденокарциноми і тривалий час мала вимушене положення в ліжку, були знайдені великі ділянки некрозу шкіри та підлеглих м’яких тканин крижової ділянки. Назвіть форму некрозу: During the autopsy of a woman who died as a result of tumor dissemination of mucinous cystadenocarcinoma and had been confined to bed for a long time, large areas of skin necrosis and underlying soft tissues were found tissues of the sacrum. Name the form of necrosis:

Секвестр Sequestration

Пролежень Decubitus bedsores

Інфаркт Heart attack

Сирнистий некроз Cheesy necrosis

Воскоподібний (ценкеровський) некроз Waxy (Zenker's) necrosis

65 / 180
На розтині чоловіка, який помер на 5-у добу захворювання черевним тифом, виявлені наступні зміни: групові фолікули клубової кишки збільшені, повнокровні і виступають над слизовою оболонкою, на їх поверхні видно борозди та звивини. Гістологічно: повнокрів’я і набряк тканини, наявність гранульом, які складаються з великих клітин зі світлою цитоплазмою і містять черевнотифозні палички. Про який період місцевих змін при черевному тифі можна думати? The autopsy of a man who died on the 5th day of typhoid fever revealed the following changes: the group follicles of the ileum are enlarged, full of blood and protrude above the mucous membrane, on furrows and convolutions are visible on their surface. Histologically: full blood and swelling of the tissue, the presence of granulomas, which consist of large cells with light cytoplasm and contain typhoid bacilli. What period of local changes in typhoid fever can we think about?

Стадія некрозу Stage of necrosis

Стадія мозкоподібного набухання Stage of cerebral swelling

Стадія чистих виразок Stage of pure ulcers

Стадія загоювання виразок Stage of ulcer healing

Стадія утворення виразок Stage of formation of ulcers

66 / 180
У клініці хворому було пересаджено нирку. Які з перерахованих клітин імунної системи можуть безпосередньо впливати на клітини трансплантату? In the clinic, a patient underwent a kidney transplant. Which of the listed cells of the immune system can directly affect the cells of the transplant?

Тимоцити Thymocytes

Плазмобласти Plasmoblasts

Т-хелпери T-helpers

Т-кілери T-killers

В-лімфоцити B-lymphocytes

67 / 180
На препараті селезінки виявляється судина, стінка якої складається з базальної мембрани з ендотелієм, середня оболонка відсутня, зовнішня оболонка зрощена зі сполучнотканинними прошарками селезінки. Що це за судина? On the preparation of the spleen, a vessel is found, the wall of which consists of a basement membrane with endothelium, the middle membrane is absent, the outer membrane is fused with the connective tissue layers of the spleen. What kind of vessel is this?

Вена м’язового типу зі слабким розвитком м’язових елементів Muscular type vein with weak development of muscular elements

Артерія м’язового типу Muscular artery

Артеріола Arteriole

Вена безм’язового типу Vein of muscleless type

Артерія еластичного типу Elastic type artery

68 / 180
У хворого спостерігається збільшений тонус артеріол за нормальних показників роботи серця. Як це вплине на величину артеріального тиску? The patient has an increased tone of arterioles with normal indicators of the heart. How will this affect the blood pressure?

Зменшиться переважно діастолічний Predominantly diastolic will decrease

Зросте переважно діастолічний Increase mainly diastolic

Зросте переважно систолічний Will increase mainly systolic

Зменшиться переважно систолічний Mainly systolic will decrease

Тиск не зміниться The pressure will not change

69 / 180
При дослідженні людини у вертикальній позі встановлено, що в альвеолах верхівок легень парціальний тиск кисню становить 140 мм рт.ст. Причиною цього є те, що у цих відділах легень: When examining a person in a vertical position, it was established that in the alveoli of the tops of the lungs, the partial pressure of oxygen is 140 mm Hg. The reason for this is that in these parts of the lungs :

Перфузія та вентиляція врівноважені Perfusion and ventilation balanced

Е. - E. -

Вентиляція відсутня No ventilation

Вентиляція переважає над перфузією Ventilation prevails over perfusion

Перфузія переважає над вентиляцією Perfusion prevails over ventilation

70 / 180
Офтальмолог з діагностичною метою (розширення зіниць для огляду очного дна) використав 1 % розчин мезатону. Мідріаз, викликаний препаратом, обумовлений: The ophthalmologist used a 1% mesaton solution for diagnostic purposes (pupil dilation to examine the fundus). Mydriasis caused by the drug is due to:

Активацією β1-адренорецепторів Activation of β1-adrenoceptors

Активацією М-холінорецепторів Activation of M-cholinergic receptors

Активацією α2-адренорецепторів Activation of α2-adrenoceptors

Активацією α1-адренорецепторів By activation of α1-adrenoceptors

Блокадою α1-адренорецепторів α1-adrenoceptor blockade

71 / 180
Чоловік 55-ти років, що скаржиться на біль в ділянці нирок, надійшов в лікарню. В ході ультразвукового обстеження пацієнта виявлено наявність ниркових каменів. Наявність в сечі якої з наведених речовин є найімовірнішою причиною утворення каменів у цього паці-єнта? A 55-year-old man, complaining of pain in the kidney area, came to the hospital. During an ultrasound examination of the patient, the presence of kidney stones was detected. The presence of which of the following in the urine which of the following substances is the most likely cause of stone formation in this patient?

Уробілін Urobilin

Білівердин Biliverdin

Білірубін Bilirubin

Сечова кислота Uric acid

Креатинін Creatinine

72 / 180
В осередку запалення утворюється біогенний амін, що має судинорозширювальну дію. Назвіть його: A biogenic amine is formed in the center of inflammation, which has a vasodilating effect. Name it:

Гістамін Histamine

Триптамін Tryptamine

ДОФА DOFA

Серотонін Serotonin

ГАМК GABA

73 / 180
При розтині померлого від чуми хворого на фоні геморагічного синдрому знайдено геморагічний некроз шкіри стегна, лімфангіт, пахвинний геморагічний лімфаденіт. Назвіть форму чуми: At the autopsy of a patient who died of the plague against the background of hemorrhagic syndrome, hemorrhagic necrosis of the skin of the thigh, lymphangitis, inguinal hemorrhagic lymphadenitis was found. Name the form of the plague:

Бубонна Bubonna

Первинно-септична Primary-septic

Первинно-легенева Primary pulmonary

Шкірно-бубонна Cutaneous bubonic

Геморагічна Hemorrhagic

74 / 180
Психологічне дослідження встановило: у людини добра здатність швидко пристосовуватися до нового оточення, добра пам’ять, емоційна стійкість, висока працездатність. Найімовірніше, ця людина: Psychological research established: a person has a good ability to quickly adapt to a new environment, good memory, emotional stability, high work capacity. Most likely, this person:

Флегматик з елементами меланхоліка Phlegmatic with elements of melancholic

Сангвінік Sanguine

Меланхолік Melancholic

Флегматик Phlegmatic

Холерик Choleric

75 / 180
Чоловік 40-ка років скаржиться на загальну слабкість, головний біль, кашель із виділенням мокротиння, задишку. Після клінічного огляду й обстеження поставлено діагноз пневмонія. Який тип гіпоксії має місце у хворого? A 40-year-old man complains of general weakness, headache, cough with sputum, shortness of breath. After a clinical examination and examination, a diagnosis of pneumonia was made. What type of hypoxia does he have the patient's place?

Гемічна Chemical

Тканинна Fabric

Гіпоксична Hypoxic

Циркуляторна Circulator

Респіраторна Respiratory

76 / 180
Хвора 45-ти років госпіталізована з підозрою на пухлину матки. Після обстеження встановлено діагноз - фіброміома матки. В якому шарі матки розміщується ця пухлина? A 45-year-old patient was hospitalized with a suspicion of a uterine tumor. After the examination, the diagnosis of uterine fibroids was established. In which layer of the uterus is this tumor located?

Perimetrium Perimeter

Parametrium Parametrium

Endometrium Endometrium

Mesometrium Mesometrium

Myometrium Myometrium

77 / 180
При дослідженні амніотичної рідини, одержаної при амніоцентезі (прокол амніотичної оболонки), виявлені клітини, ядра яких містять статевий хроматин (тільце Барра). Про що з зазначеного це може свідчити? In the study of amniotic fluid obtained during amniocentesis (puncture of the amniotic membrane), cells were found, the nuclei of which contain sex chromatin (Barr body). Which of the above can this indicate? testify?

Генетичні порушення в розвитку плода Genetic disorders in fetal development

Трисомія Trisomy

Поліплоїдія Polyploidy

Розвиток плода чоловічої статі Male fetus development

Розвиток плода жіночої статі Development of a female fetus

78 / 180
У хворої 43-х років після чергового загострення ревмокардиту з’явилися ознаки декомпенсації серцевої діяльності з виникненням набряків на ногах і асциту. Затримці води в організмі хворої сприяло підвищення продукції: A 43-year-old patient, after another exacerbation of rheumatic carditis, showed signs of decompensation of cardiac activity with swelling of the legs and ascites. Water retention in the patient's body was facilitated by increased production :

Кортикотропіну Corticotropin

Кортизолу Cortisol

Тироксину Thyroxine

Інсуліну Insulin

Альдостерону Aldosterone

79 / 180
У хворого спостерігається порушення зору - гемералопія ('куряча сліпота'). Який вітамінний препарат треба вживати хворому, щоб відновити зір? The patient has visual impairment - hemeralopia ('chicken blindness'). What vitamin preparation should the patient take to restore vision?

Токоферолу ацетат Tocopherol Acetate

Піридоксин Pyridoxine

Вікасол Vikasol

Тіаміну хлорид Thiamine chloride

Ретинолу ацетат Retinol Acetate

80 / 180
Хвора 38-ми років надійшла в реанімаційне відділення в непритомному стані. Рефлекси відсутні. Цукор крові - 2,1 ммоль/л. В анамнезі: цукровий діабет з 18-ти років. Яка кома має місце у хворої? A 38-year-old patient was admitted to the intensive care unit in an unconscious state. There are no reflexes. Blood sugar - 2.1 mmol/l. History: diabetes since 18 years old. What kind of coma does the patient have?

Кетоацидотична Ketoacidotic

Гіперглікемічна Hyperglycemic

Гіпоглікемічна Hypoglycemic

Лактацидемічна Lactacidemic

Гіперосмолярна Hyperosmolar

81 / 180
У збудливій клітині заблокували іонні канали, внаслідок чого клітина з часом повністю втратила потенціал спокою. Які канали заблокували? Ion channels were blocked in an excitable cell, as a result of which the cell completely lost its resting potential over time. Which channels were blocked?

Калієві Potassium

Калієві та натрієві Potassium and sodium

Кальцієві Calcium

Хлорні Chlorni

Натрієві Sodium

82 / 180
У збудливій клітині заблокували іонні канали. Це не змінило суттєво рівень потенціалу спокою, але клітина втратила здатність до генерації ПД. Які канали заблокували? Ion channels were blocked in an excitable cell. This did not significantly change the resting potential level, but the cell lost its ability to generate PD. Which channels were blocked?

Кальцієві Calcium

Натрієві Sodium

Калієві Potassium

Натрієві та калієві Sodium and potassium

Хлорні Chlorni

83 / 180
У хворого діагностовано пухлину мозку, яка розміщена в ділянці 'пташиної шпори'. Порушення якої функції виникне у хворого, якщо пухлина буде активно розвиватися? The patient has been diagnosed with a brain tumor, which is located in the area of the 'bird's spur'. Which function will be impaired in the patient if the tumor is actively developing?

Зір Sight

Смак Taste

Слух Hearing

Нюх Sniff

Дотикова чутливість Touch sensitivity

84 / 180
У хворого виявлено порушення периферичного кровообігу, основою якого є обмеження припливу артеріальної крові. При цьому має місце збліднення даної ділянки, зниження місцевої температури. Це обумовлене: The patient has a violation of peripheral blood circulation, the basis of which is the limitation of arterial blood flow. At the same time, there is a pallor of this area, a decrease in local temperature. This is due to:

Сладж-феноменом Sludge phenomenon

Ішемією Ischemia

Артеріальною гіперемією Arterial hyperemia

Лімфостазом Lymphostasy

Венозною гіперемією Venous hyperemia

85 / 180
Хворому 35-ти років для обстеження очного дна був призначений атропіну сульфат у вигляді очних крапель. Для відновлення акомодації йому закрапали пілокарпіну гідрохлорид, але це не дало бажаного ефекту Що є причиною відсутності ефекту? A 35-year-old patient was prescribed atropine sulfate in the form of eye drops for examination of the fundus. To restore accommodation, he was instilled with pilocarpine hydrochloride, but this did not have the desired effect. What is the reason for the lack of effect?

Двосторонній антагонізм Bilateral antagonism

Односторонній антагонізм One-sided antagonism

Синергізм Synergism

Звикання Addiction

Тахіфілаксія Tachyphylaxis

86 / 180
В процесі експерименту на собаці виникла необхідність підвищити збудливість серцевого м’язу За допомогою введення якого іону можливо досягти бажаного ефекту? In the course of an experiment on a dog, it became necessary to increase the excitability of the heart muscle. With the help of the introduction of which ion is it possible to achieve the desired effect?

Са2+ Ca2+

С1~ С1~

К+ С Fe2+ К+ С Fe2+

Mg2+ Mg2+

87 / 180
Гістони - ядерні білки, які пов’язані з ДНК іонним зв’язком. Які амінокислоти обумовлюють основний характер гісто- нів? Histones are nuclear proteins that are connected to DNA by an ionic bond. What amino acids determine the basic character of histones?

Аргінін і лізин Arginine and Lysine

Пролін і метіонін Proline and methionine

Серин і цистеїн Serine and Cysteine

Іліцин та аланін Ilicine and Alanine

Лейцин та ізолейцин Leucine and Isoleucine

88 / 180
Для діагностування деяких хромосомних хвороб використовують визначення статевого хроматину. Назвіть хворобу, при якій потрібне це визначення: Sex chromatin determination is used to diagnose some chromosomal diseases. Name the disease that requires this determination:

Хвороба Дауна Down's disease

Синдром Шерешевського-Тернера Shereshevsky-Turner syndrome

Гемофілія Hemophilia

Синдром Патау Patau syndrome

Хвороба Брутона Bruton's disease

89 / 180
До лікаря звернулась жінка зі скаргами на те, що після операції видалення щитоподібної залози у неї спостерігається порушення мови, захриплість. Пошкодження якого нерва під час операції могло стати причиною цього явища? A woman turned to the doctor with complaints that after the operation to remove the thyroid gland, she has a speech disorder, hoarseness. Damage to which nerve during the operation could have caused this phenomena?

Язиково-глоткового нерва Linguo-pharyngeal nerve

Під’язикового нерва Hypoglossal nerve

Зворотного гортанного нерва Reverse laryngeal nerve

Верхнього гортанного нерва Upper laryngeal nerve

Язикового нерва Lingual nerve

90 / 180
У жінки, хворої на злоякісну пухлину молочної залози, лікар виявив збільшення регіонарних лімфатичних вузлів. Яка група вузлів підлягає ураженню при даній патології в першу чергу? In a woman suffering from a malignant tumor of the breast, the doctor found an increase in regional lymph nodes. Which group of nodes is subject to damage in this pathology in the first place?

Ліктьові вузли Elbow Nodes

Пахвинні вузли Inguinal nodes

Шийні вузли Cervical nodes

Діафрагмальні вузли Diaphragmatic nodes

Пахвові вузли Axillary nodes

91 / 180
До хірургічного відділення ЦРЛ надійшов хворий з колотою раною стопи, яку він отримав під час косовиці. Які специфічні препарати необхідно застосувати з метою екстреної активно-пасивної імунопрофілактики правця? A patient was admitted to the surgical department of the Central Hospital with a puncture wound of the foot, which he received during a bloodbath. What specific drugs should be used for the purpose of emergency active-passive immunoprophylaxis of tetanus?

Протиправцева вакцина Tetanus vaccine

Вакцина АКДП AKDP vaccine

Антитоксична сироватка та анатоксин Antitoxic serum and toxoid

Антибіотики Antibiotics

Інтерферон Interferon

92 / 180
У п’ятимісячної дівчинки виявлено застійні явища у легенях. При обстеженні виявлено зв’язок між висхідною аортою та легеневою артерією, що в нормі спостерігається у деяких земноводних і плазунів. Назвіть цю природжену ваду розвитку: A five-month-old girl was found to have congestion in the lungs. The examination revealed a connection between the ascending aorta and the pulmonary artery, which is normally observed in some amphibians and reptiles . Name this congenital malformation:

Транспозиція магістральних судин Transposition of trunk vessels

Розвиток правої дуги аорти Development of the right aortic arch

Дефект міжпередсердної перегородки Atrial septal defect

Дефект міжшлуночкової перегородки Ventricular septal defect

Незрощення боталової протоки Nonunion of the botal duct

93 / 180
Лікар-дослідник у складі альпіністської експедиції піднявся у базовий табір, розташований на висоті 5000 м. На 3-й день перебування у нього з’явилися ознаки гірської хвороби: задишка, головний біль, втрата апетиту, загальна слабкість, ціаноз. Який тип гіпоксії має місце в цьому випадку? A research doctor as part of a mountaineering expedition climbed to the base camp, located at an altitude of 5000 m. On the 3rd day of his stay, he developed symptoms of mountain sickness: shortness of breath, headache, loss of appetite, general weakness, cyanosis. What type of hypoxia occurs in this case?

Тканинна Fabric

Змішана Mixed

Гіпоксична Hypoxic

Циркуляторна Circulator

Іемічна Iemic

94 / 180
У чоловіка 30-ти років перед операцією визначили групову належність крові. Кров резус-позитивна. Реакцію аглютинації еритроцитів не викликали стандартні сироватки груп 0оД (І), АЗ (II), Beu (ПІ). Досліджувана кров належить до групи: The blood group of a 30-year-old man was determined before the operation. The blood is Rh-positive. The erythrocyte agglutination reaction was not caused by standard sera of groups 0oD (I), AZ ( II), Beu (PI). The studied blood belongs to the group:

АД (II) AD (II)

Во (III) In (III)

AB (IV) AB (IV)

ОоД (І) OoD (I)

- -

95 / 180
Локальне порушення кровопостачання міокарда призвело до порушення у клітинах синтезу АТФ та до припинення роботи натрій-калієвих насосів. Найбільш імовірним наслідком цього буде така зміна потенціалу спокою клітин у зоні пошкодження: Local disruption of the blood supply to the myocardium led to disruption of ATP synthesis in the cells and to the cessation of sodium-potassium pumps. The most likely consequence of this will be such a change in the resting potential of cells in the area of damage :

Зникнення Disappearance

Суттєве збільшення Significant increase

Збільшення Increase

Несуттєве зменшення Insignificant decrease

Відсутність змін No changes

96 / 180
На аутопсії тіла жінки, що хворіла на хронічну дизентерію, в ході гістологічного дослідження внутрішніх органів в стромі та паренхімі міокарда, нирок, в слизовій оболонці шлунка та в сполучній тканині легень виявлені аморфні відкладення фіолетового кольору, що дають позитивну реакцію за Коссом. Яке ускладнення розвинулося у хворої? At the autopsy of the body of a woman suffering from chronic dysentery, during the histological examination of internal organs in the stroma and parenchyma of the myocardium, kidneys, in the mucous membrane of the stomach and in the connective tissue purple amorphous deposits were found in the lungs, giving a positive reaction according to Koss. What complication developed in the patient?

Метаболічне звапніння Metabolic calcification

Гіаліноз Hyalinosis

Амілоїдоз Amyloidosis

Метастатичне звапніння Metastatic calcification

Дистрофічне звапніння Dystrophic calcification

97 / 180
Розпад глікогену в печінці стимулюється адреналіном. Який вторинний ме- сенджер (посередник) при цьому утворюється в клітині? Decomposition of glycogen in the liver is stimulated by adrenaline. What secondary messenger is formed in the cell?

CO D.NO CO D.NO

Діацилгліцерол Diacylglycerol

ц-ГМФ ts-HMF

ц-АМФ ts-AMP

98 / 180
Дитина 3-х років із симптомами стоматиту, гінгівіту, дерматиту відкритих ділянок шкіри була госпіталізована. При обстеженні встановлено спадкове порушення транспорту нейтральних амінокислот у кишечнику. Нестачею якого вітаміну будуть зумовлені дані симптоми? A 3-year-old child with symptoms of stomatitis, gingivitis, dermatitis of open areas of the skin was hospitalized. During the examination, a hereditary disorder of the transport of neutral amino acids in the intestines was found. The lack of which vitamin will are these symptoms caused?

Біотину Biotin

Ніацину Niacin

Вітаміну А Vitamin A

Пантотенової кислоти Pantothenic acid

Кобаламіну Cobalamin

99 / 180
Хворий ходить хитаючись, широко розставляючи ноги. У нього знижений тонус м’язів рук і ніг, скандована мова. У якому відділі головного мозку локалізується ураження? The patient walks unsteadily, spreading his legs wide. He has reduced tone of the muscles of the arms and legs, slurred speech. In which part of the brain is the lesion localized?

Моторна кора Motor cortex

Червоне ядро Red core

Мозочок Cerebellum

Хвостате ядро Tail kernel

Шкаралупа Shell

100 / 180
Встановлено, що токсична дія ціанідів виявляється у гальмуванні клітинного дихання. Який органоїд клітини є найбільш чутливим до цих отрут? It has been established that the toxic effect of cyanides is manifested in inhibition of cellular respiration. Which cell organoid is the most sensitive to these poisons?

Клітинний центр Cell Center

Мітохондрїї Mitochondria

Лізосоми Lysosomes

Комплекс Іольджі Iolji Complex

Рибосоми Ribosomes

101 / 180
Юнак 17-ти років страждає на фурункульоз, викликаний умовно- патогенним Staphylococcus epidermidis. Яке дослідження найдоцільніше провести, щоб правильно вибрати препарат для лікування цього хворого? A 17-year-old boy suffers from furunculosis caused by opportunistic Staphylococcus epidermidis. What research is most appropriate to conduct in order to choose the right drug for the treatment of this patient?

Визначити фаговар Define Phagovar

Визначити антигенні властивості Determine antigenic properties

Дослідити біохімічні властивості Explore biochemical properties

Виявити фактори патогенності Identify pathogenicity factors

Скласти антибіотикограму Make an antibiotic chart

102 / 180
У хворого впродовж 10-ти днів має місце підвищена температура, напади характерного кашлю. Лікар призначив посів слизу з носоглотки на середовище казеїнова-вугільний агар. Який мікроорганізм передбачається виявити? For 10 days, the patient has a high temperature, attacks of a characteristic cough. The doctor prescribed a culture of mucus from the nasopharynx on a casein-carbon agar medium. What microorganism is expected to be detected ?

Лістерію Listeria

Клебсіелу Klebsiel

Паличку інфлюенци Influenza

Стафілокок Staphylococcus

Паличку коклюшу Whooping cough

103 / 180
Хворому призначена ендоскопія 12- палої кишки. В результаті виявлено запалення великого дуоденального сосочка і порушення виділення жовчі в просвіт кишки. У якому відділі 12-палої кишки виявлені порушення? The patient is scheduled for endoscopy of the duodenum. As a result, inflammation of the large duodenal papilla and violation of the secretion of bile into the lumen of the intestine were detected. In which part of the duodenum were violations detected?

Іоризонтальна частина Horizontal part

Верхня частина Top

Цибулина Onion

Висхідна частина Eastern part

Низхідна частина Descending part

104 / 180
У дитини, яка часто хворіє на ангіни та фарингіти, відзначається збільшення лімфовузлів і селезінки. Зовнішній вигляд характеризується пастозністю та блідістю, м’язова тканина розвинена слабко. У крові спостерігається лімфо- цитоз. Як називається такий вид діатезу? In a child who often suffers from sore throats and pharyngitis, an increase in lymph nodes and spleen is noted. The appearance is characterized by pastiness and paleness, muscle tissue is poorly developed. In the blood lymphocytosis is observed. What is the name of this type of diathesis?

Лімфатико-гіпопластичний Lymphatic-hypoplastic

Нервово-артритичний Nervous-arthritic

Ексудативно-катаральний Exudative-catarrhal

Геморагічний Hemorrhagic

Астенічний Asthenic

105 / 180
У жінки 35-ти років, яка протягом З місяців обмежувала кількість продуктів у харчовому раціоні, спостерігається зменшення маси тіла, погіршення фізичного стану та розумової діяльності, з’явилися набряки. Дефіцит яких харчових речовин міг призвести до таких змін? A 35-year-old woman, who limited the number of products in her diet for 3 months, observed a decrease in body weight, deterioration of physical condition and mental activity, appeared edemas. Deficiency of what nutrients could have led to such changes?

Вуглеводів Carbs

Вітамінів Vitamins

Жирів Fat

Білків Proteins

Мікроелементів Microelements

106 / 180
Хворому діагностовано недостатність мітрального клапана. Де даний клапан розміщується? The patient is diagnosed with mitral valve insufficiency. Where is this valve located?

Між лівим і правим шлуночками Between the left and right ventricles

Між правим передсердям і правим шлуночком Between the right atrium and the right ventricle

Між лівим і правим передсердями Between the left and right atria

В місці виходу аорти At the exit of the aorta

Між лівим передсердям і лівим шлуночком Between the left atrium and the left ventricle

107 / 180
При проведенні оперативного втручання на шиї хірургу потрібно виділити зовнішню сонну артерію. Що є анатомічним орієнтиром для встановлення місця початкового відділу вказаної судини в ділянці шиї? When performing surgery on the neck, the surgeon needs to isolate the external carotid artery. What is the anatomical reference point for determining the location of the initial section of the specified vessel in the neck?

Нижній край щитоподібного хряща Inferior edge of thyroid cartilage

Яремна вирізка Jog tenderloin

Місце початку грудинно-ключично- соскоподібного м’яза Place of origin of the sternocleidomastoid muscle

Кут нижньої щелепи Angle of the lower jaw

Верхній край щитоподібного хряща Upper border of thyroid cartilage

108 / 180
У 18-річної дівчини в сільському районі Індії розвився профузний пронос з втратою рідини до 8 літрів на добу. Який з наведених нижче мікроорганізмів може бути збудником захворювання? An 18-year-old girl in rural India developed profuse diarrhea with fluid loss of up to 8 liters per day. Which of the following microorganisms can be the causative agent of the disease?

Ентеропатогенна Escherichia coli Enteropathogenic Escherichia coli

Salmonella typhi Salmonella typhi

Campylobacter jejuni Campylobacter jejuni

Shigella dysenteriae Shigella dysenteriae

Vibrio cholerae Vibrio cholerae

109 / 180
Хворий госпіталізований з попереднім діагнозом 'черевний тиф'. Яке живильне середовище з перерахованих можна використати для виділення гемо- культури? The patient is hospitalized with a preliminary diagnosis of typhoid fever. Which nutrient medium from the list can be used to isolate a blood culture?

Жовтково-сольовий агар Yolk-salt agar

Кров ’яний агар Blood agar

МПБ MPB

Середовище Левенштейна-Иенсена Lewenstein-Jensen environment

Жовчний бульйон Bile broth

110 / 180
При обстеженні лікар встановив, що дисфункція кори головного мозку пацієнта викликана ураженням мережі нейронів мозкового стовбура, яка підтримувала активність кори великих півкуль. Які структури мозку уражені? During the examination, the doctor established that the dysfunction of the patient's cerebral cortex was caused by damage to the network of neurons in the brainstem, which supported the activity of the cortex of the large hemispheres. What brain structures are affected?

Ядра мозочка Cerebellar nuclei

Ретикулярна формація Reticular Formation

Ядра гіпоталамуса Nuclei of the hypothalamus

Базальні ядра Basal cores

Ядра таламуса Thalamus nuclei

111 / 180
У пацієнта перфоративна виразка передньої стінки шлунка. В яке похідне очеревини потрапить вміст шлунка? The patient has a perforating ulcer of the anterior wall of the stomach. Into which derivative of the peritoneum will the contents of the stomach enter?

Правий брижовий синус Right mesenteric sinus

Лівий брижовий синус Left mesenteric sinus

Передшлункова сумка Pregastric pouch

Печінкова сумка Liver bag

Чепцева сумка Cap Bag

112 / 180
У результаті радіаційного випромінювання були ушкоджені стовбурові гемопоетичні клітини. Утворення яких клітин сполучної тканини буде порушено? Hematopoietic stem cells were damaged as a result of radiation. The formation of which connective tissue cells will be disturbed?

Макрофаги Macrophages

Фібробласти Fibroblasts

Перицити Pericytes

Меланоцити Melanocytes

Адипоцити Adipocytes

113 / 180
Реалізація загального адаптаційного синдрому здійснюється переважно через нейроендокринну систему. Якій з ланок цієї системи належить провідна роль у патогенезі реакції, що розвивається? The implementation of the general adaptation syndrome is carried out mainly through the neuroendocrine system. Which of the links of this system has the leading role in the pathogenesis of the developing reaction?

Гіпофізарно-юкстагломерулярна Pituitary-juxtaglomerular

Гіпофізарно-адреногенітальна Pituitary-adrenogenital

Гіпофізарно-адреналова Pituitary-adrenal

Гіпофізарно-інсулярна Pituitary-insular

Гіпофізарно-тиреоїдна Pituitary-thyroid

114 / 180
У відділення реанімації надійшов хворий після ДТП з однобічним пневмотораксом. Який вид дихання спостерігається у цьому випадку? A patient was admitted to the intensive care unit after a road accident with unilateral pneumothorax. What type of breathing is observed in this case?

Поверхневе рідке Surface liquid

Глибоке часте Deep Frequent

Асфіктичне Asphyxic

Поверхневе Superficial

Поверхневе часте Surface frequent

115 / 180
У хворого зі скаргами на полідипсію, поліфагію, поліурію визначили високий рівень глюкози крові і наявність її у сечі. Яке захворювання можна припустити? In a patient with complaints of polydipsia, polyphagia, polyuria, a high level of blood glucose and its presence in urine were determined. What disease can be assumed?

Акромегалія Acromegaly

Нецукровий діабет Diabetes insipidus

Цукровий діабет Diabetes

Хвороба Аддісона Addison's disease

Інсулінома Insulinoma

116 / 180
Хворому на активну форму туберкульозу призначили ізоніазид. Який вітамінний препарат необхідно застосовувати для профілактики побічної дії ізоніазиду? The patient was prescribed isoniazid for an active form of tuberculosis. What vitamin preparation should be used to prevent side effects of isoniazid?

Ретинолу ацетат Retinol Acetate

Ціанокобаламін Cyanocobalamin

Рутин Routine

Піридоксину гідрохлорид Pyridoxine hydrochloride

Токоферолу ацетат Tocopherol Acetate

117 / 180
Відомо, що однією з причин виникнення мембранного потенціалу спокою є різниця концентрації іонів по обидві сторони клітинної мембрани. Який механізм забезпечує іонну асиметрію всередині і зовні клітини? It is known that one of the reasons for the resting membrane potential is the difference in the concentration of ions on both sides of the cell membrane. What mechanism ensures ion asymmetry inside and outside the cell?

Полегшена дифузія Diffusion facilitated

Фільтрування Filtering

Дифузія Diffusion

Піноцитоз Pinocytosis

Активний транспорт Active transport

118 / 180
Людина застосовувала дихальну методику, яка заснована на проведенні частого та поверхнього дихання, що супроводжується гіповентиляцією. Внаслідок чого при цьому розвивається респіраторний ацидоз? The person used a breathing technique that is based on frequent and shallow breathing accompanied by hypoventilation. As a result, respiratory acidosis develops?

- -

Нестачі О2 в крові Lack of O2 in the blood

Надлишку О2 в крові Excess O2 in blood

Нестачі СО2 в крові Lack of CO2 in the blood

Надлишку С*О2 в крові Excess C*O2 in blood

119 / 180
Під дією УФ-опромінення та інших факторів можуть відбуватися зміни в структурі ДНК. Репарація молекули ДНК досягається узгодженою дією всіх наступних ферментів, ЗА ВИНЯТКОМ: Under the influence of UV-irradiation and other factors, changes in the structure of DNA can occur. Repair of the DNA molecule is achieved by the coordinated action of all the following enzymes, EXCEPT:

Ендонуклеази Endonucleases

Аміноацил-тРНК-синтетази Aminoacyl-tRNA synthetases

ДНК-глікозид ази DNA Glycoside Az

ДНК-лігази DNA ligases

ДНК-полімерази DNA polymerases

120 / 180
Батько і син мають різні мітохондрі- альні геноми. Це пояснюється тим, що ці геноми: Father and son have different mitochondrial genomes. This is because these genomes:

Мають високий рівень мутабільності Have a high level of mutability

Успадковуються від матері Inherited from mother

Мають різну експресивність Have different expressiveness

Підлягають комбінативній мінливості Subject to combinatorial variability

Не успадковуються Not inherited

121 / 180
У жінки, що тривало приймала антибіотики з приводу кишкової інфекції, розвинулось ускладнення з боку слизової порожнини рота у вигляді запального процесу і білого нальоту, у якому під час бактеріологічного дослідження були виявлені дріжджеподібні грибки Candi-da albicans. Який з перерахованих препаратів показаний для лікування цього ускладнення? A woman who had been taking antibiotics for an intestinal infection for a long time developed a complication from the mucous membrane of the mouth in the form of an inflammatory process and a white plaque, in which during a bacteriological examination yeast-like fungi Candi-da albicans were detected. Which of the listed drugs is indicated for the treatment of this complication?

Поліміксин Polymixin

Флуконазол Fluconazole

Тетрациклін Tetracycline

Фуразолідон Furazolidone

Бісептол Biseptol

122 / 180
Хвора 57-ми років для лікування гіпертонічної хвороби тривалий час приймала анаприлін. Побічні ефекти спонукали пацієнтку відмовитись від прийому препарату, що призвело до розвитку гіпертонічного кризу і нападу стенокардії. Як називається ускладнення, яке виникло? A 57-year-old patient took anaprilin for a long time to treat hypertension. The side effects prompted the patient to stop taking the drug, which led to the development of a hypertensive crisis and an attack of angina pectoris. What is the name of the complication that occurred?

Звикання Addiction

Лікарська залежність Drug addiction

Синдром відміни Withdrawal Syndrome

Сенсибілізація Sensitization

Тахіфілаксія Tachyphylaxis

123 / 180
При деяких гельмінтозах людина може сама виявити гельмінта, оскільки зрілі членики збудника можуть активно виповзати з ануса людини. Це характерно для: With some helminthiasis, a person can detect the helminth himself, as mature members of the pathogen can actively crawl out of the anus of a person. This is characteristic of:

Дифілоботріозу Diphyllobotriosis

Теніаринхозу Taeniarhynchosis

Ехінококозу Echinococcosis

Теніозу Taeniosis

Гіменолепідозу Hymenolepidosis

124 / 180
Жінка літнього віку перенесла сильний стрес. У крові різко збільшилась концентрація адреналіну і норадреналі- ну. Які ферменти каталізують процес інактивації катехоламінів? An elderly woman suffered severe stress. The concentration of adrenaline and noradrenaline in the blood increased sharply. What enzymes catalyze the process of catecholamine inactivation?

Пептидази Peptidases

Ілікозидази Ilicosidases

Моноамінооксидази Monoamine oxidases

Карбоксилаза Carboxylase

Тирозиназа Tyrosinase

125 / 180
У чоловіка, померлого від внутрішньої кровотечі (гемоперитонеум), в печінці субкапсулярно виявлено губчастий вузол темно-червоного кольору розмірами 15x10 см, добре відмежований від навколишньої тканини. Мікроскопічно: тканина вузла складається з великих судинних тонкостінних порожнин, вистелених ендотеліальними клітинами та заповнених рідкою або згорнутою кров’ю. Встановіть вид пухлини: In a man who died of internal bleeding (hemoperitoneum), a dark red spongy nodule measuring 15x10 cm was found in the liver subcapsularly, well separated from the surrounding tissue. Microscopically: the tissue of the node consists of large vascular thin-walled cavities lined with endothelial cells and filled with liquid or coagulated blood. Establish the type of tumor:

Кавернозна гемангіома Cavernous hemangioma

Лімфангіома Lymphangioma

Венозна гемангіома Venous hemangioma

Іемангіоперицитома Hemangiopericytoma

Капілярна гемангіома Capillary hemangioma

126 / 180
На заняттях з лікувальної фізкультури лікар-фізіотерапевт запропонував юнакам відхилитися назад і дістати долонями до підлоги. Яка зв’язка запобігає надмірному розгинанню хребтового стовпа? During a physical therapy class, a physiotherapist suggested the young men lean back and touch the floor with their palms. What ligament prevents excessive extension of the spinal column?

Жовта Yellow

Міжпоперечна Intertransverse

Надостьова Nadostova

Передня поздовжня Front longitudinal

Задня поздовжня Back Longitudinal

127 / 180
Вивчається мітотичний поділ клітин епітелію ротової порожнини. Встановлено, що в клітині диплоїдний набір хромосом. Кожна хромосома складається з двох максимально спіралізованих хроматид. Хромосоми розташовані у площині екватору клітини. Така картина характерна для стадії мітозу: The mitotic division of the cells of the oral cavity epithelium is being studied. It was established that the cell has a diploid set of chromosomes. Each chromosome consists of two maximally spiraled chromatids. The chromosomes are located in the plane of the cell's equator. Such a picture is typical for the stage of mitosis:

Анафаза Anaphase

Метафаза Metaphase

Профаза Prophase

Телофаза Telophase

Прометафаза Prometaphase

128 / 180
У хворого з хронічним гіперацидним гастритом з’явився біль у суглобах. Для полегшення болю, враховуючи супутню патологію, був призначений целекоксиб. Вибіркова дія цього препарату на певний фермент забезпечує відсутність впливу на слизову шлунка. Назвіть цей фермент: A patient with chronic hyperacid gastritis developed pain in the joints. To alleviate the pain, taking into account the accompanying pathology, celecoxib was prescribed. The selective effect of this drug on a certain enzyme provides no effect on the gastric mucosa Name this enzyme:

Фосфоліпаза А2 Phospholipase A2

Фосфоліпаза С Phospholipase C

Калікреїн Kallikrein

Циклооксигеназа 2 Cyclooxygenase 2

Циклооксигеназа 1 Cyclooxygenase 1

129 / 180
Після травми печінки у хворого з’явились симптоми отруєння аміаком за типом печінкової коми. Як аміак діє на енергозабезпечення ЦНС? After a liver injury, the patient developed symptoms of ammonia poisoning according to the type of hepatic coma. How does ammonia affect the energy supply of the central nervous system?

Інактивація ферментів дихального ланцюга Inactivation of respiratory chain enzymes

Іальмування бета-окислення жирних кислот Inhibition of beta-oxidation of fatty acids

Іальмування гліколізу Ialmation of glycolysis

Блокування ЦТК внаслідок зв’язування альфа-кетоглутарату CTC blocking due to alpha-ketoglutarate binding

Інгібування окисного фосфорилюван- ня Inhibition of oxidative phosphorylation

130 / 180
Бактеріологічний метод діагностики був використаний для підтвердження діагнозу газова гангрена у хворого. Які живильні середовища необхідно використовувати для культивування збудника в цьому випадку? The bacteriological method of diagnosis was used to confirm the diagnosis of gas gangrene in the patient. What nutrient media should be used for the cultivation of the pathogen in this case?

Вільсона-Блера, Кітта-Тароцці Wilson-Blair, Kitt-Tarozzi

МПА, МПБ MPA, MPB

Лужний агар Alkaline Agar

Ендо, Левіна, Плоскірєва Endo, Levina, Ploskireva

ЖСА, кров’яний агар ZHA, blood agar

131 / 180
У дівчинки 6-ти років виражені ознаки гемолітичної анемії. При біохімічному аналізі еритроцитів встановлено дефіцит ферменту глюкозо-6- фосфатдегідрогенази. Порушення якого метаболічного процесу відіграє головну роль у розвитку цієї патології? A 6-year-old girl has pronounced signs of hemolytic anemia. A biochemical analysis of erythrocytes revealed a deficiency of the enzyme glucose-6-phosphate dehydrogenase. Disruption of this metabolic process plays a major role in the development of this pathology?

Окисного фосфорилювання Oxidative phosphorylation

Глюконеогенезу Gluconeogenesis

Пентозофосфатного шляху Pentose phosphate pathway

Тканинного дихання Tissue breathing

Анаеробного гліколізу Anaerobic glycolysis

132 / 180
Скорочення поперечно-посмугованих м’язів неможливе без кальцію. Яку роль відіграє цей іон в утворенні актино- міозинових містків? The contraction of striated muscles is impossible without calcium. What role does this ion play in the formation of actin-myosin bridges?

З’єднується з тропоніном Bounds to troponin

З’єднується з адренорецепторами Connects with adrenoceptors

З’єднується з серотоніновими рецепторами Connects to serotonin receptors

З’єднується з холінорецептором Connects to cholinergic receptor

З’єднується з гістаміновими рецепторами Connects to histamine receptors

133 / 180
У хворого з синдромом Іценка- Кушинга спостерігаються стійка гіперглікемія та глюкозурія. Синтез та секреція якого гормону підвищені у цього хворого? A patient with Itsenko-Cushing syndrome has persistent hyperglycemia and glucosuria. Which hormone's synthesis and secretion are increased in this patient?

Глюкагон Glucagon

Альдостерон Aldosterone

Адреналін Adrenaline

Кортизол Cortisol

Тироксин Thyroxine

134 / 180
Обмеження споживання води призвело до зневоднення організму. Який механізм активується для збереження води в організмі? Restriction of water consumption led to dehydration of the body. What mechanism is activated to conserve water in the body?

Зменшення секреції кальцитоніну Decreased calcitonin secretion

Зменшення секреції альдостерону Decreased secretion of aldosterone

Збільшення секреції тироксину Increased secretion of thyroxine

Збільшення секреції соматостатину Increased secretion of somatostatin

Збільшення секреції вазопресину Increased vasopressin secretion

135 / 180
У хворого на подагру виявлено значне підвищення рівня сечової кислоти в крові. Кінцевим продуктом обміну яких речовин є сечова кислота? In a gout patient, a significant increase in the level of uric acid in the blood was detected. What metabolic end product is uric acid?

Альбумінів Albumin

Пуринових основ Purine bases

Тригліцеридів Triglycerides

Глобулінів Globulins

Жирних кислот Fatty acids

136 / 180
При тривалому використанні одного з гормональних препаратів у хворого виявлено ознаки остеопорозу, ерозії слизової шлунка, гіперглікемію, зменшення рівня АКТГ в крові. Препарат якої групи гормонів міг викликати ці явища? During long-term use of one of the hormonal drugs, the patient showed signs of osteoporosis, erosion of the gastric mucosa, hyperglycemia, and a decrease in the level of ACTH in the blood. The drug of which group of hormones could cause these phenomena ?

Кальцитоніну Calcitonin

Ілюкокортикоїдів Ileucocorticoids

Мінералокортикоїдів Mineralocorticoids

Статевих гормонів Sex hormones

Иодвмісних гормонів Iodine hormones

137 / 180
Людина потрапила в ситуацію, що пов’язана з емоційним напруженням. У результаті цього в неї в крові підвищився рівень адреналіну і, як наслідок, збільшилась сила серцевих скорочень. Яким чином адреналін збільшує силу серцевих скорочень? A person got into a situation related to emotional stress. As a result, the level of adrenaline in his blood increased and, as a result, the force of heart contractions increased. How does adrenaline increase the force of heart contractions?

Впливає на барорецептори Affects baroreceptors

Знижує збудливість клітин- пейсмекерів Reduces excitability of pacemaker cells

Збуджує бета-адренорецептори Stimulates beta-adrenoceptors

Викликає збудження барорецепторів Causes baroreceptors to be excited

Знижує тонус блукаючих нервів Reduces the tone of vagus nerves

138 / 180
У хворого на малярію після вживання протималярійного препарату при- махіну розвинулася гемолітична анемія. Спадкова недостатність якого ферменту в еритроцитах спостерігається при цьому? A patient with malaria developed hemolytic anemia after taking the antimalarial drug primachin. Hereditary deficiency of which enzyme in erythrocytes is observed?

Фосфофруктокінази Phosphofructokinase

Ілюкозо-6-фосфатдегідрогенази Ilucose-6-phosphate dehydrogenase

Фруктозо-1-фосфатальдолази Fructose-1-phosphate aldolase

Ліпази Lipase

Тріозофосфатізомерази Triose phosphate isomerases

139 / 180
Під час проведення морфологічного дослідження периферичної крові хворого було помічено, що у еритроцитів забарвлена лише периферична частина, а в центрі є незабарвлене прояснення. Кольоровий показник - 0,56. Яка анемія найімовірніша у цього пацієнта? During the morphological examination of the patient's peripheral blood, it was noticed that only the peripheral part of the erythrocytes is stained, and there is a colorless clearing in the center. The color index is 0.56. What anemia is most likely in this patient?

Ві2 фолієводефіцитна Vi2 folic acid deficiency

Апластична Aplastic

Сидеробластна Syderoblastna

Залізодефіцитна Iron deficiency

Іемолітична Hemolytic

140 / 180
На фельдшерський пункт звернулась жінка з приводу рваної рани правої кисті, що була спричинена власним мисливським собакою, який загинув через 5 днів від сказу. Які препарати потрібно використати для профілактики сказу у вкушеної жінки? A woman came to the medical center with a laceration on her right hand, which was caused by her own hunting dog, which died 5 days later from rabies. What drugs should be used for prevention rabies in a bitten woman?

Антирабічна вакцина+антирабічний імуноглобулін Anti-rabies vaccine+anti-rabies immunoglobulin

Вакцина БЦЖ BCG vaccine

Інтерферон Interferon

Антирабічна сироватка-і-антибіотики Anti-rabies-serum-and-antibiotics

Антибіотики Antibiotics

141 / 180
У хворого, який звернувся в шкірно- венерологічний диспансер, виявлений твердий шанкр. Яке мікробіологічне дослідження дозволить поставити діагноз сифілісу на даному етапі хвороби? A solid chancre was found in a patient who went to a dermatovenerological dispensary. What microbiological examination will allow a diagnosis of syphilis at this stage of the disease?

Темнопольна мікроскопія Dark field microscopy

Алергологічний Allergological

Бактеріологічний Bacteriological

Серологічний Serological

Біологічний Biological

142 / 180
У фермера, що має стадо кіз, спостерігається лихоманка неясного походження. Найімовірніше, збудником захворювання є: A farmer with a herd of goats has a fever of unknown origin. The most likely cause of the disease is:

Brucella melitensis Brucella melitensis

Т pallidum T pallidum

S. aureus S. aureus

Histoplasma capsulatum Histoplasma capsulatum

Clostridium novyi Clostridium novyi

143 / 180
Жінка 35-ти років розпочала голодування. Депо яких поживних речовин використовується у початковий період голодування і як при цьому змінюється дихальний коефіцієнт (ДК)? A 35-year-old woman started fasting. What nutrient depot is used in the initial period of fasting and how does the respiratory coefficient (DC) change?

Білки, ДК наближається до 0,7 Proteins, DC approaching 0.7

Білки, ДК наближається до 1 Proteins, DC approaching 1

Жири, ДК наближається до 0,85 Fats, DC approaching 0.85

Жири, ДК наближається до 0,72 Fats, DC approaching 0.72

Вуглеводи, ДК наближається до 1 Carbs, DC approaching 1

144 / 180
У хворого діагностовано паразитарне захворювання, яке супроводжується клінічною картиною В12- фолієводефіцитної анемії. Який гельмінт спричинив дане захворювання? The patient was diagnosed with a parasitic disease, which is accompanied by the clinical picture of B12 foliodeficiency anemia. What helminth caused this disease?

Fasciola hepatica Fasciola hepatica

Hymenolepis nana Hymenolepis nana

Taenia solium Taenia solium

Taeniarhynchus saginatus Taeniarhynchus saginatus

Diphyllobothrium latum Diphyllobothrium latum

145 / 180
У хворого з підозрою на озену з носоглотки були виділені грамнегатив- ні палички, які утворювали капсулу на поживному середовищі. Які мікроорганізми спричинили хворобу? Gram-negative bacilli were isolated from the nasopharynx of a patient suspected of ozena, which formed a capsule on a nutrient medium. What microorganisms caused the disease?

Мікоплазми Mycoplasma

Хламідії Chlamydia

Сальмонели Salmonella

Шигели Schigels

Клебсієли Klebsiels

146 / 180
Чоловік з гострим міокардитом помер від серцево-судинної недостатності. В ході мікроскопічного дослідження внутрішніх органів виявлені: плазмора- гія, набряк, стази в капілярах, численні крововиливи, а також дистрофічні зміни в паренхімі. Наслідком чого є дані зміни? A man with acute myocarditis died of cardiovascular failure. Microscopic examination of internal organs revealed: plasmarrhagia, edema, capillary stasis, numerous hemorrhages, and as well as dystrophic changes in the parenchyma. What are the consequences of these changes?

Хронічний загальний венозний застій Chronic general venous stasis

Загальне артеріальне повнокрів’я Total arterial complete blood

ДВЗ -синдром DVZ syndrome

Місцеве артеріальне повнокрів’я Local arterial complete blood

Гострий загальний венозний застій Acute general venous stasis

147 / 180
При складних методах фарбування виявляють особливості хімічного складу бактеріальної клітини або наявність певних структур. Який з перерахованих методів фарбування є основним та найчастіше вживаним? With complex staining methods, peculiarities of the chemical composition of the bacterial cell or the presence of certain structures are revealed. Which of the listed staining methods is the main and most often used?

Ожешко Ozheshko

Леффлера Leffler

Грама Gram

Романовського-Гімзи Romanovsky-Giemza

Ціля-Нільсена Nielsen Target

148 / 180
Пацієнтці 68-ми років з туберкульозом легень призначено антибіотик, прийом якого може викликати забарвлення сечі та сльозної рідини в червоний колір. Назвіть його: A 68-year-old patient with pulmonary tuberculosis has been prescribed an antibiotic that can cause urine and tear fluid to turn red. Name it:

Тетрациклін Tetracycline

Етіонамід Ethionamide

Рифампіцин Rifampicin

Етамбутол Etambutol

Амоксиклав Amoxiclav

149 / 180
У жінки під час мейозу відбулося порушення розходження аутосом. Утворилася яйцеклітина із зайвою 18-ю хромосомою. Яйцеклітина запліднюється нормальним сперматозооном. У майбутньої дитини буде синдром: During meiosis, a woman had a violation of autosome segregation. An egg cell with an extra 18th chromosome was formed. The egg cell is fertilized by a normal spermatozoon. The future child will have the syndrome:

Патау Patau

Едвардса Edwards

Шерешевського-Тернера Shereshevsky-Turner

Дауна Down

Клайнфельтера Klinefelter

150 / 180
У клініку госпіталізовано хворого з діагнозом карцинома кишечника. Для більшості карцином характерна підвищена продукція і секреція серотоніну. Відомо, що ця речовина утворюється з триптофану в ході реакції: A patient with a diagnosis of intestinal carcinoma was hospitalized in the clinic. Most carcinomas are characterized by an increased production and secretion of serotonin. It is known that this substance is formed from tryptophan during the reaction:

Дезамінування Demining

Трансамінування Transamination

Декарбоксилювання Decarboxylation

Мікросомального окислення Microsomal oxidation

Утворення парних сполук Formation of paired compounds

151 / 180
After severe emotional strain a 45-year-old man suddenly developed constricting pain in the area of his heart. The pain was irradiating into his left arm, neck, and left shoulder blade. His fase was pale and covered in cold sweat. Nitroglycerine was able to relieve the pain. What condition developed in the patient in this case? Після сильного емоційного перенапруження у чоловіка 45 років раптово з'явився стискаючий біль у ділянці серця. Біль віддавав у ліву руку, шию, і лівої лопатки. Тіло бліде, вкрите холодним потом. Нітрогліцерин зміг зняти біль. Який стан розвинувся у хворого в цьому випадку?

Angina pectoris Стенокардія

Myocardial infarction Інфаркт міокарда

Gastric ulcer perforation Перфорація виразки шлунка

Psychogenic shock Психогенний шок

Stroke Інсульт

152 / 180
A 23-year-old woman was brought into the emergency department complaining of bloody diarrhea, tiredness, and dizziness. A few days ago she went to a fast food reataurant for a birthday party. Her friends are experiencing similar symptoms. Laboratory studies show anemia. What samples should be obtained for microbiologic testing in this case? 23-річна жінка була доставлена у відділення невідкладної допомоги зі скаргами на криваву діарею, втому та запаморочення. Кілька днів тому вона пішла до фаст-фуду ресторан на день народження. Подібні симптоми спостерігаються у її знайомих. Лабораторні дослідження показують анемію. Які проби взяти для мікробіологічного дослідження в цьому випадку?

Stool Табурет

Cerebrospinal fluid Спинномозкова рідина

Urine Сеча

Bile Жовч

Blood Кров

153 / 180
Histological microslide shows a gastrointestinal organ. The wall of this organ consists of 4 layers: mucosal, submucosal, muscular, and serous. The muscular layer has folds and pits. What organ has such appearance? Гістологічний мікропрепарат показує орган шлунково-кишкового тракту. Стінка цього органу складається з 4 шарів: слизового, підслизового, м'язового та серозного. М'язовий шар має складки та ямки .Який орган має такий вигляд?

Appendix Додаток

Stomach Шлунок

Small intestine Тонка кишка

Duodenum Дванадцятипала кишка

Esophagus Стравохід

154 / 180
A 2-year-old child with delayed physical and mental development was brought to the hospital. The child’s parents are the most concerned by frequent profuse vomiting that occurs in their child after eating. Laboratory testing detected phenylpyruvic acid in the child’s urine. What type of metabolism is disturbed, causing this patology? До лікарні доставлена дитина 2-х років із затримкою фізичного та розумового розвитку. Найбільше занепокоєння батьків дитини викликає часте рясне блювання, яке виникає у дитини після їжі. При лабораторному дослідженні в сечі дитини виявлена фенілпіровиноградна кислота. Який вид обміну речовин порушений, що спричинило дану патологію?

Phosphorus and calcium metabolism Обмін фосфору та кальцію

Lipid metabolism Ліпідний обмін

Amino acid metabolism Метаболізм амінокислот

Carbohydrate metabolism Вуглеводний обмін

Water and electrolyte metabolism Метаболізм води та електролітів

155 / 180
The heart is a muscular organ that pumps blood through the body. This function is carried out by valves, muscles, and vessels. What valve is located between the left ventricle and left atrium? Серце - це м'язовий орган, який перекачує кров тілом. Цю функцію виконують клапани, м'язи та судини. Який клапан розташований між лівим шлуночка і лівого передсердя?

Non-return Без повернення

Tricuspid Тристулковий

Bicuspid Двозубий

Aortic Аорта

Pulmonary Легеневий

156 / 180
During your physiology class, the professor asks you to explain the effect of various hormones and neurotransmitters on the metabolism of glucose in the human body. You open your report with the statement that the use of glucose by the cells is preceded by its transport from the intercellular substance into the cell. What hormone is most likely responsible for the glucose uptake by the cell? Під час вашого уроку фізіології професор просить вас пояснити вплив різних гормонів і нейромедіаторів на метаболізм глюкози в організмі людини. Ви починаєте свою доповідь за допомогою твердження, що використанню глюкози клітинами передує її транспортування з міжклітинної речовини в клітину. Який гормон, швидше за все, відповідає за поглинання глюкози клітиною?

Adrenaline Адреналін

Aldosterone Альдостерон

Thyroxine Тироксин

Insulin Інсулін

Glucagon Глюкагон

157 / 180
A 25-year-old man came to the family doctor complaining of the loss of taste sensation in the front two- thirds of his tongue. The doctor determined that this condition was caused by the damage to a certain nerve. What nerve is likely to be functionally impaired in this case? 25-річний чоловік звернувся до сімейного лікаря зі скаргами на втрату смакових відчуттів у передніх двох третинах язика. Лікар визначив, що цей стан був викликаний пошкодженням певного нерва. Який нерв, ймовірно, буде функціонально порушений у цьому випадку?

Accessory nerve Додатковий нерв

Vagus Вагус

Hypoglossal nerve Під'язиковий нерв

Chorda tympani Chorda tympani

Glossopharyngeal nerve Язикоглотковий нерв

158 / 180
A 28-year-old man complains of nausea, vomiting, and right-sided subcostal pain. Objectively, his skin and sclerae are icteric, he has elevated body temperature, enlarged liver, dark urine, fecal hypocholia, hyperbilirubinemia (direct and indirect bilirubin), bilirubinuria, urobilinuria, hypoproteinemia, and decreased blood coagulability. What condition can be characterized by these changes? Чоловік 28 років скаржиться на нудоту, блювоту, біль у правому підребер'ї. Об'єктивно: шкіра та склери жовтяничні, тіло піднесене. температура, збільшення печінки, темна сеча, гіпохолія калу, гіпербілірубінемія (прямий і непрямий білірубін), білірубінурія, уробілінурія, гіпопротеїнемія, зниження згортання крові. Який стан характеризує ці зміни?

Acute pancreatitis Гострий панкреатит

Pre-hepatic hemolytic jaundice Печінкова гемолітична жовтяниця

Acute cholecystisis Гострий холецистиз

Hepatocellular parenchymal jaundice Печінково-клітинна паренхіматозна жовтяниця

Post-hepatic jaundice Постпечінкова жовтяниця

159 / 180
A soft tissue node obturated the patient’s middle lobe bronchus, leading to the development of right middle lobe atelectasis. In the area of obturation, bronchial biopsy detected proliferations of atypical glandular epithelium with pathological mitoses. Epithelium penetrates into the underlaying tissues and cartilage. What disease is the most likely in this case? М'якотканинний вузол обтурував середньодолевий бронх пацієнта, що призвело до розвитку ателектазу правої середньої частки. У ділянці обтурації біопсія бронха виявила проліферації атипового бронха залозистий епітелій з патологічними мітозами. Епітелій проникає в підлягаючі тканини та хрящі. Яке захворювання найбільш імовірне в даному випадку?

Bronchial epithelial dysplasia Бронхіальна епітеліальна дисплазія

Bronchial sarcoma Бронхіальна саркома

Inflammatory polyp Запальний поліп

Bronchogetic lung cancer Бронхогетичний рак легенів

Bronchitis deformans Деформуючий бронхіт

160 / 180
The main functions of neural tissue can be described as communication and integration. What is the basic unit of neural tissue that ensures its functioning? Основні функції нервової тканини можна описати як зв'язок та інтеграція. Яка основна одиниця нервової тканини забезпечує її функціонування?

Neuron Нейрон

Axon Аксон

Nucleus Ядро

Myelin sheath Мієлінова оболонка

Dendrite Дендрит

161 / 180
The patient has developed a refractive error, where the focal point of the image is located not precisely on the retina, but anterior to it. This causes distant objects to be blurry, while close objects often appear normal. Among the other signs there are headaches and eye strain. This condition is associated with retinal detachment, cataract, and glaucoma. Name this condition: У пацієнта виникла аномалія заломлення, коли фокусна точка зображення розташована не саме на сітківці, а попереду неї. Це спричиняє віддалені об’єкти бути розмитим, тоді як близькі предмети часто здаються нормальними. Серед інших ознак – головний біль і напруга очей. Цей стан пов’язаний із відшаруванням сітківки, катарактою та глаукомою. Назвіть цей стан:

Emmetropia Емметропія

Astigmatism Астигматизм

Farsightedness Далекозорість

Nearsightedness Короткозорість

Blindness Сліпота

162 / 180
A 20-year-old woman came to the clinic after missing her last 2 periods. Her cycles are usually regular, occurring at 28-30 day interval with moderate bleeding and occasional abdominal pain. She also complains of progressively diminishing peripheral vision. Her doctor determined the loss of vision in the lateral halves of both retinas. What structure is likely to be alfected, causing bitemporal hemianopsia in the patient? 20-річна жінка звернулася до клініки після відсутності останніх 2 менструацій. Її цикли зазвичай регулярні, з інтервалом 28-30 днів із помірним кровотеча та періодичні болі в животі. Вона також скаржиться на прогресуюче погіршення периферичного зору. Її лікар визначив втрату зору в бічних половинах обох сітківок. Яка структура, ймовірно, уражена, що спричиняє бітемпоральну геміанопсію у пацієнта?

Optic chiasm Оптичний хіазм

Right optic nerve Правий зоровий нерв

Left optic nerve Лівий зоровий нерв

Right optic tract Правий зоровий тракт

Left optic tract Лівий зоровий тракт

163 / 180
A woman was bitten by a venomous snake (Macrovipera lebetinis). Autopsy of her body shows marked intravascular hemolysis. Her pancreas, bone marrow, and lymph nodes are brown. Microscopy detects abundant brown pigment in the macrophage cytoplasm. What pigment accumulates in the tissues in such cases? Жінку вкусила отруйна змія (Macrovipera lebetinis). Розтин її тіла показує виражений внутрішньосудинний гемоліз. Її підшлункова залоза, кістковий мозок і лімфатичні вузли коричневі .Під час мікроскопії в цитоплазмі макрофагів виявляється велика кількість коричневого пігменту. Який пігмент накопичується в тканинах у таких випадках?

Lipofuscin Ліпофусцин

Hemosiderin Гемосидерин

Hematoidin Гематоідин

Hematin Гематин

Bilirubin Білірубін

164 / 180
Histological microslide shows an organ consisting of gray and white matter. The gray matter is located in the center and is made up of bundle neurons (neurocytus funicularis), radicular neurons (neurocytus radiculatus), and interneurons. Such morphology is characteristic of the following organ? Гістологічний мікропрепарат показує орган, що складається з сірої та білої речовини. Сіра речовина розташована в центрі та складається з нейронів пучка (neurocytus funicularis), корінцевого нейрони (neurocytus radiculatus), інтернейрони. Така морфологія характерна для наступного органу?

Cerebral hemispheres Півкулі головного мозку

Cerebellum Мозочок

Spinal ganglion Спинномозковий ганглій

Medulla oblongata Довгастий мозок

Spinal cord Спинний мозок

165 / 180
A molecular biologist studies various molecules. One of them is a polymeric molecule essential in various biological roles in coding, decoding, regulation, and expression of genes. It is a helical single­stranded molecule folded onto itself. Which of the following molecules is being studied? Молекулярний біолог вивчає різні молекули. Однією з них є полімерна молекула, яка відіграє важливу роль у різних біологічних ролях у кодуванні, декодуванні, регуляції та експресії генів. Це спіральна одноланцюгова молекула, згорнута сама на себе. Яка з наведених молекул досліджується?

HLA HLA

DNA ДНК

RNA РНК

ADP ADP

ATP ATP

166 / 180
There is a system that maintains the balanced state of multicellular organisms. Its main functions are to ensure an adequate physiological response and to defend the body against infectious agents and other unintended invasions. What system has such functions? Існує система, яка підтримує збалансований стан багатоклітинних організмів. Її основні функції полягають у забезпеченні адекватної фізіологічної реакції та захисті організму від інфекційних агентів та інших ненавмисне вторгнення. Яка система має такі функції?

Lymphatic system Лімфатична система

Immune system Імунна система

Homeostatic system Гомеостатична система

Nervous system Нервова система

Endocrine system Ендокринна система

167 / 180
A team of medical students researches the phases of cell cycle. During one of the mitotic phases the cell has nearly completed its division, the chromosomes decondense, and two nuclei begin to form around them. What phase is likely observed in the cell at this moment? Команда студентів-медиків досліджує фази клітинного циклу. Під час однієї з мітотичних фаз клітина майже завершила свій поділ, хромосоми деконденсуються, і два ядра навколо них починають формуватися. Яка фаза ймовірно спостерігається в клітині в цей момент?

Anaphase Анафаза

Metaphase Метафаза

Prophare Prophare

Telophase Телофаза

168 / 180
A 34-yeat-old man visits his dentist complaining of a toothache. After a dental procedure that involved extraction of several teeth, he developed a severe bleeding lasting more than 15 minutes. He has a history of chromic hepatitis C. What is the most likely cause of the prolonged bleeding in this patient? 34-річний чоловік звернувся до стоматолога зі скаргами на зубний біль. Після стоматологічної процедури, яка передбачала видалення кількох зубів, у нього почалася сильна кровотеча, яка тривала більше більше 15 хв.. У нього в анамнезі хромний гепатит С. Яка найбільш вірогідна причина тривалої кровотечі у цього пацієнта?

Thrombocytopenia Тромбоцитопенія

Hypoalbuminemia Гіпоальбумінемія

Hypocalcemia Гіпокальціємія

Hypofibrinogenemia Гіпофібриногенемія

169 / 180
Medical examination in an army recruitment center detected a 15-year-old boy with tall stature, eunuchoid body proportions, gynecomastia, and female pattern of pubic hain growth. The boy has fat deposits on the thighs, no facial fair growth, high-pitched voice, and below average IQ. He was diagnosed with Klinefelter syndrome. What karyotype corresponds with this disease? Медичне обстеження в армійському пункті призову виявило 15-річного хлопчика високого зросту, євнухоїдних пропорцій тіла, гінекомастії та жіночого типу росту лобка. У хлопчика жирові відкладення на стегнах, рябого росту обличчя немає, голос високий, IQ нижче середнього. Діагностований синдром Клайнфельтера. Який каріотип відповідає цьому захворюванню?

47, XYY 47, XYY

47, XY, +18 47, XY, +18

45, X0 45, X0

46, XY 46, XY

47, XXY 47, XXY

170 / 180
A man with signs of peritonitis was brought to the admission room. He has a 12-year-long history of peptic ulcer disease with ulcer localization on the posterior gastric wall. He was diagnosed with gasiric ulcer perforation. What anatomical structure is likely to be contaminated with gastric content in this case? У приймальне відділення доставлений чоловік з ознаками перитоніту. У нього 12 років виразкова хвороба з локалізацією виразки на задніх відділах шлунка. стінки. Діагностовано: перфорація виразки шлунка. Яка анатомічна структура ймовірно забруднена шлунковим вмістом у даному випадку?

Canalis lateralis dexter Canalis lateralis dexter

Bursa omentalis Сальникова сумка

Canalis lateralis sinister Canalis lateralis sinister

Bursa pregastrica Предшлункова сумка

Bursa hepatica Bursa hepatica

171 / 180
A 46-year-old man complains of tiredness and pain in the joints of his fingers and wrists. These signs are observed for the last 2 months. The pain is present in both hands and the wrists are swollen. Furthermore, he describes morning stiffness in his joints, lasting about 2 hours, which improves with use. His past medical history reveals that he was successfully treated for H. pylori - related ulcer last year. He denies smoking and stopped drinking when his GI symptoms started. What drug is the best choice for his joint pain management? Чоловік 46 років скаржиться на втомлюваність і біль у суглобах пальців і зап'ястя. Ці ознаки спостерігаються протягом останніх 2 місяців. Біль присутній в обох руках, а зап’ястя опухлі. Крім того, він описує ранкову скутість у суглобах, яка триває приблизно 2 години, яка покращується під час використання. Його минула медична історія показує, що минулого року він успішно лікувався від виразки, пов’язаної з H. pylori .Він заперечує куріння та кинув пити, коли у нього почалися симптоми шлунково-кишкового тракту. Який препарат є найкращим вибором для лікування болю в суглобах?

Aspirin Аспірин

Prednisolone Преднізолон

Celecoxib Целекоксиб

Patacelanol Патацеланол

Morphine Морфін

172 / 180
A 27-year-old woman complains of insomnia, irritability, hand tremor, acute weight loss despite high appetite, and constant fever with body temperature of 37,2-37,5 Co. What endocrine gland is likely to be functionally impaired in this case? Жінка 27 років скаржиться на безсоння, дратівливість, тремор рук, різку втрату ваги, незважаючи на високий апетит, постійну лихоманку з температурою тіла 37,2. -37,5 Ко. Функціональне порушення якої залози внутрішньої секреції в даному випадку ймовірно?

Thyroid gland Щитовидна залоза

Neurohypophysis Нейрогіпофіз

Pancreas Підшлункова залоза

Parathyroid gland Паращитовидна залоза

Adrenal glands Надниркові залози

173 / 180
General practitioner performs physical examination of the patient. In the course of the examination it is necessary to measure the palpable regular expansion of a superficial artery caused by the ejection of blood into the vessels through heart contractions. What sign does the doctor measure? Лікар загальної практики проводить фізикальний огляд пацієнта. Під час огляду необхідно виміряти пальпаторне регулярне розширення поверхневої артерії, викликане викидом кров в судини через серцеві скорочення Яку ознаку вимірює лікар?

Blood pressure Кров'яний тиск

Saturaion Насиченість

Heartbeat Серцебиття

Heart electricity Електрика серця

Pulse Пульс

174 / 180
A man has signs of acute respiratory viral disease. His physician referred him for microbiological testing. A mucus smear obtained trom the patient’s tonsils contains spherical microorganisins arranged in short chains. The patient was diagnosed with tonsillitis. What microorganisms were detected in the smear from the patient's tonsils? У чоловіка ознаки гострого респіраторного вірусного захворювання. Лікар направив його на мікробіологічне дослідження. У мазку слизу з мигдаликів пацієнта містяться сферичні мікроорганізми, розташовані короткими ланцюжками .У хворого діагностовано тонзиліт.Які мікроорганізми були виявлені в мазку з мигдалин у хворого?

Streptocci Streptocci

Diplococci Diplococci

Staphylococci Стафілококи

Tetracocci Тетракокки

Micrococci Мікрококи

175 / 180
Muscle relaxant tubocurarine was administered during a surgery. After the surgery the patient's breathing is not restored. What antidote should he be given to resore his breathing? Під час операції введено міорелаксант тубокурарин. Після операції дихання пацієнта не відновлюється. Який антидот йому слід дати, щоб відновити дихання?

Proserin (Neostigmine) Прозерин (неостигмін)

Clophelin (Clonidine) Клофелін (клонідин)

Atropine sulfate Атропіну сульфат

Bemegride Bemegride

Anaprilin (Propranolol) Анаприлін (пропранолол)

176 / 180
A 45-yeat-old woman came to her physician with complaints of extreme tiredness and weakness. She says that these symptoms lasts for a month already. Within the last 2 weeks she losts 3 kilograms. Objectively, she is tired-looking thin woman. Skin hyperpigmentation is observed in many areas of her body, most prominently on the face, neck, and the backs of her hands (the areas exposed to light). What hormone is produccd in an excess in this patient, most likely causing the hyperpigmentation? Жінка 45 років звернулася до лікаря зі скаргами на сильну втому та слабкість. Вона каже, що ці симптоми тривають уже місяць. Протягом останнього За 2 тижні схудла на 3 кілограми.Об'єктивно худа жінка втомленого вигляду.Гіперпігментація шкіри спостерігається на багатьох ділянках тіла, найбільше на обличчі, шиї та тильній стороні рук (ділянках, які потрапляють під світло). Який гормон виробляється в надлишку у цього пацієнта, що, швидше за все, викликає гіперпігментацію?

Gonadotropins Гонадотропіни

Growth hormone (GH) Гормон росту (GH)

P-Lipotopin P-ліпотопін

Melanocyte-stimulating hormone (MSH) Меланоцитстимулюючий гормон (МСГ)

Thyroid-stimulating hormone (TSH) Тиреотропний гормон (ТТГ)

177 / 180
A 50-year-old woman complains of constant thist. She drinks large amounts of liquid and has increased diuresis. Her blood glucose is 12 mmol/L. Her urine contains glucose. What endocrine organ is likely to be functionally impaired in this patient? Жінка 50 років скаржиться на постійне це. П'є велику кількість рідини, посилений діурез. Рівень глюкози в крові 12 ммоль/л. сеча містить глюкозу Функціональне порушення якого ендокринного органу у даного пацієнта ймовірно?

Adrenal glands Надниркові залози

Thyroid gland Щитовидна залоза

Pancreas Підшлункова залоза

Parathyroid gland Паращитовидна залоза

Neurohypophysis Нейрогіпофіз

178 / 180
An unidentified surgical specimen is received fot histopathologic analysis. A portion of the specimen is cut and stained with hematoxylin and eosin. Under the microscope, you see an organ encapaulated by dense connective tissue that extends to the deeper areas by way of the trabecular extensions. The organ can be subdivided into two regions: a cortex with lymphoid nodules and medulla with medullary cords populated by plasma cells, B cells, and Tcells. What anatomical structure is the most likely origin of this surgical specimen? Неідентифікований хірургічний зразок отримано для гістопатологічного аналізу. Частину зразка вирізають і фарбують гематоксиліном і еозином. Під мікроскопом ви бачите інкапсульований орган щільною сполучною тканиною, яка поширюється на глибші ділянки за допомогою трабекулярних розширень. Орган можна розділити на дві частини: кору з лімфоїдними вузликами та мозкову речовину з мозковими тяжами, населеними плазматичними клітинами, В-клітинами та Т-клітинами. Яка анатомічна структура найбільш ймовірне походження цього хірургічного зразка?

Thymus Тимус

Lymph node Лімфатичний вузол

Tonsils Мигдалини

Spleen Сплін

Bone marrow Кістковий мозок

179 / 180
A 65-year-old woman was brought into the emergency departmnent because of shortness of breath and chest pain that started a few hours ago. She denied having fever, expectoration, or any accompanying symptoms. She had a 5-year-long history of deep vein thrombosis in her thin. After a time, she died of respiratory distress. Autopsy reveals red loose masses lodged in the bifurcation of the pulmonary trunk with extensions into both the left and the right pulmonary arteries. Whnt is the most likely diagnosis? 65-річна жінка була доставлена у відділення невідкладної допомоги через задишку та біль у грудях, які почалися кілька годин тому. Вона заперечувала, що у неї лихоманка, відхаркування або будь-які супутні симптоми. У неї був 5-річний анамнез тромбозу глибоких вен у худий. Через деякий час вона померла від респіраторного дистресу. Розтин показує червоні пухкі утворення, розташовані в біфуркації легеневого стовбура з розширеннями в як ліва, так і права легенева артерія. Whnt найбільш імовірний діагноз?

Myocardial infarction Інфаркт міокарда

Pneumonia Пневмонія

Thromboembolism Тромбоемболія

Pneumothorax Пневмоторакс

180 / 180
A 54-year-old woman had a total thyroidectomy lor papillary thyroid carcinoma. 11 hours after operation she complained of tingling around her mouth. On physical examination, the Trousseau's sign and Chvostek's sign are present. Her condition rapidly deteriorates with laryngospasm and focal seizures. The surgeon suspects surgical destruction of the parathyroid glands. What is the most likely cause of this patient's neurological condition? 54-річній жінці була проведена тотальна тиреоїдектомія або папілярна карцинома щитовидної залози. Через 11 годин після операції вона скаржилася на поколювання навколо рота. Під час фізичного огляду Наявні ознаки Труссо та Хвостека. Її стан швидко погіршується з ларингоспазмом та фокальними судомами. Хірург підозрює хірургічне руйнування паращитовидних залоз. Яка найбільш ймовірна причина неврологічного стану цієї пацієнтки?

Hypocalcemia Гіпокальціємія

Hyperkalemia Гіперкаліємія

Hyperchloremia Гіперхлоремія

Hypophosphatemia Гіпофосфатемія

Hyponatremia Гіпонатріємія